NPTE Practice Questions

Pataasin ang iyong marka sa homework at exams ngayon gamit ang Quizwiz!

December 24, 2020 A patient describes a number of symptoms in their fingers and hands consistent with Raynaud's disease. Which patient would have the greatest incidence of this condition? 1) 29 female 2) 57 female 3) 27 male 4) 55 male

1) 29 female

February 27, 2021 A physical therapist treats a patient status post MI. The physician prescribes moderate exercises no greater than 5 METs. Which interpretation is most consistent with this MET level? 1) 5x the energy at rest 2) 5x the breaths per minute at rest 3) 5x the forced expiratory volume at rest 4) 5x the tidal volume at rest

1) 5x the energy at rest

April 16, 2020 A physical therapist presents an in-service on the risk factors associated with developing osteoporosis. Which of the following patients would be the most at risk for developing osteoporosis? 1) 67-year-old w/ malabsorption syndrome 2) 70-year-old w/ metabolic alkalosis 3) 68-year-old male with fibromyalgia 4) 72-year-old male with cauda equina syndrome

1) 67-year-old w/ malabsorption syndrome

June 19, 2020 A young, healthy adult is breathing at tidal volume. What percentage of the tidal volume reaches the respiratory units and is available for gas exchange? 1) 70% 2) 80% 3) 90% 4) 100%

1) 70%

June 21, 2020 A patient is examined by a physician after a recent onset of hand and finger weakness and muscular fasciculations. During the physician visit, the patient is informed that their medical condition is very serious and they have 2-5 years to live. Which diagnosis is the most consistent with this scenario? 1) ALS 2) MS 3) Parkinson's disease 4) Huntington's disease

1) ALS

March 27, 2021 A patient with a complete SCI has use of the wrist extensors on the R arm and use of the elbow extensors on the L arm. Sensory information is intact up to the C5 dermatome on both sides. What is the appropriate neurological level assigned to this patient? 1) C5 2) C6 3) C7 4) C8

1) C5

December 8, 2020 A physical therapist evaluates a patient using the Modified Ashworth Scale and documents a rating 3. Which of the following conditions would most likely result in a patient exhibiting a rating of 3 on this scale? 1) CP 2) down syndrome 3) DMD 4) Guillain-Barre syndrome

1) CP

Dec 4, 2019 A physical therapist is reviewing a medical record of a patient with pulmonary pathology. The lungs show destruction of the bronchioles in the upper lobes with inflammation and thickened bronchiolar walls. The patient's history includes 30 years of smoking. The clinical presentation is most likely associated with which of the following medical diagnosis? 1) Centrilobular emphysema 2) Paraseptal emphysema 3) Pneumonia 4) Pulmonary fibrosis

1) Centrilobular emphysema

April 27, 2020 A patient with hip joint pain is referred to physical therapy for direction in weight bearing strengthening exercises. The physician's referral notes that recent bone density testing revealed the patient has osteopenia. Which of the following co-morbidities would most likely be part of the patient's medical history? 1) Cushing's syndrome 2) Gullain-Barre syndrome 3) spinal stenosis 4) hypoparathyroidism

1) Cushing's syndrome

Dec 6, 2019 A physical therapist consults with a speech-language pathologist regarding a patient who has abulic aphasia. Based on the type of presenting aphasia, which of the following impairments would be most anticipated? 1) Delayed response time to questions 2) Limited speech production with jumbled words 3) Inability to understand questions being asked 4) Inability to produce or comprehend language

1) Delayed response time to questions

Jan 5, 2020 A physical therapist employed in an acute hospital attempts to identify the current status of a patient following a TBI. Which objective piece of documentation in the chart would suggest the most severe brain injury? 1) Glascow coma scale: 4 2) Glascow coma scale: 14 3) levels of cognitive functioning scale, level IV 4) levels of cognitive functioning scale, level VIII

1) Glascow coma scale: 4

May 30, 2020 A physical therapist observes a patient attempt to walk on their heels as part of a lower quarter screening. What would this activity likely assess? 1) L4 myotome 2) L5 myotome 3) S1 myotome 4) S2 myotome

1) L4 myotome

September 3, 2020 A physical therapist reviews a case report on a recently deceased patient. The case report indicates that the patient died during the third trimester of pregnancy secondary to a pre-existing condition. What is this scenario most consistent with? 1) Marfan syndrome 2) Addison's disease 3) Guillain-Barre syndrome 4) osteogenesis imperfecta

1) Marfan syndrome

March 23, 2021 A PT treats a patient with a tibial plateau fracture that is currently partial weight bearing. The PTA is educating the patient for stairs using bilateral canes, and the patient falls. Who is most liable for the incident? 1) PT 2) PTA 3) patient 4) referring physician

1) PT

April 18, 2020 A patient with cardiac arrhythmias has been prescribed beta-blockers. What is the most effective way to monitor the patient's activity tolerance? 1) RPE 2) VAS 3) palpation of radial heart rate 4) assessment of telemetry strip

1) RPE

September 14, 2020 A patient reports to physical therapy with a referral for a TENS unit. The patient is a 42-year-old male with lengthy history of back pain. During the physical therapy session the patient asks if he can use the TENS when driving his car. What is the most appropriate response? 1) TENS can be used while driving, but be alert 2) TENS can be used while driving, but limit to 1 hour a day 3) avoid using while driving since it will delay reaction time 4) avoid using while driving since it may produce involuntary muscle contractions

1) TENS can be used while driving, but be alert

April 28, 2020 A physical therapist examines a patient who reports lateral elbow pain that the therapist suspects is due to epicondylitis. Which of the following patient profiles would be most expected based on the diagnosis? 1) a 42-year old patient with unilateral symptoms 2) a 50-year-old patient with bilateral symptoms 3) a 64-year-old patient with unilateral symptoms 4) a 73-year-old patient with bilateral symptoms

1) a 42-year old patient with unilateral symptoms

January 3, 2020 A patient on prolonged bed rest attempts to get out of bed. Upon attaining a standing position, the patient complains of lightheadedness and blurred vision. What is the most likely cause of the patient's symptoms? 1) a decrease in BP 2) a decrease in RR 3) an increase in HR 4) an adverse reaction to medication

1) a decrease in BP

March 31, 2020 A physical therapist observes a patient perform a sit to stand transfer and notices that the patient begins the transfer with their feet positioned anteriorly compared to the typical foot placement. This modification would most likely result in which of the following outcomes? 1) a greater duration of LE extension once the thigh leaves the supporting surface during the transfer 2) a lesser duration of LE extension once the thigh leaves the supporting surface during the transfer 3) a greater magnitude of hip extension motion to initiate the transfer and move the body mass over the feet 4) a lesser magnitude of hip flexion motion to initiate the transfer and move the body mass over the feet

1) a greater duration of LE extension once the thigh leaves the supporting surface during the transfer

November 5, 2020 A physical therapist performs an examination on a patient diagnosed with piriformis syndrome. The patient reports experiencing pain in the low back and buttock region for the last three weeks. Which motions should the therapist expect to be weak and painful during muscle testing based on the patient's diagnosis? 1) abd and lateral rotation of the thigh 2) abd and medial rotation of the thigh 3) add and lateral rotation of the thigh 4) add and medial rotation of the thigh

1) abd and lateral rotation of the thigh

Nov 25, 2019 A physical therapist observes a patient utilizing a hip strategy to shift the center of mass during a balance test. Which muscle group would be activated first during forward sway using this postural strategy? 1) abdominals 2) plantar flexors 3) paraspinals 4) dorsiflexors

1) abdominals

March 12, 2020 A patient 10 days following a TKA returns to physical therapy after a follow-up visit with their surgeon. The patient reports being excited since they were told to prematurely discontinue use of the compression garments shown in the image. Which variable would most likely have influenced the surgeon's decision? 1) activity level 2) knee range of motion 3) use of pain medication 4) postoperative time frame

1) activity level

December 14, 2020 A patient with bilateral transfemoral amputation presents to PT for WC mobility training on an inclined surface. What modification is the most appropriate to reduce the WC from becoming unstable during ramp ascension? 1) addition of posterior anti-tippers 2) addition of anterior anti-tippers 3) recline WC 10 degrees posteriorly 4) tilt the WC posteriorly by changing the front caster alignment

1) addition of posterior anti-tippers

October 8, 2020 A physcial therapist treats a patient with multiple sclerosis. The patient does not have a history of seizures, however, has been prescribed an anticonvulsant medication to assist in the treatment of neuropathic pain. What would this form of treatment best described as? 1) adjuvant medication 2) immunomodulating medication 3) interventional medicine 4) alternative medicine

1) adjuvant medication

June 24, 2020 A physical therapist examines a patient with fibromyalgia. The patient reports that contact with the bed sheets produces pain when he tries to sleep at night. What is this finding most consistent with? 1) allodynia 2) hyperalgesia 3) hypesthesia 4) analgesia

1) allodynia

March 18, 2020 A physical therapist works on balance reactions with a patient who has Charcot's foot. Which of the following findings is most likely related to the development of this medical condition? 1) altered sensation 2) muscular weakness 3) ulcer formation 4) pes cavus

1) altered sensation

March 19, 2021 A physical therapist administers the capillary refill test to a patient with a neurovascular compression injury. What is the most important component to assess when identifying a positive or negative test? 1) amount of time take for color to return to nail 2) amount of time taken to create blanching of the nail 3) amount of color returning to nail 4) amount of blanching of the nail

1) amount of time take for color to return to nail

May 16, 2020 After being discharged from an acute rehabilitation hospital, a patient diagnosed with complete L4 paraplegia is referred to outpatient physical therapy. The patient's primary goal is to ambulate community distances using crutches. Which orthosis will the patient most likely require to meet this goal? 1) ankle-foot orthosis 2) knee-ankle-foot orthosis 3) drop lock knee-ankle-foot orthosis 4) hip-knee-ankle-foot orthosis

1) ankle-foot orthosis

Feb 20, 2020 A physical therapist is treating a patient with cystic fibrosis in an outpatient setting. The patient has a history of GERD and is currently being treated with Prevacid. When performing postural drainage, the treatment of which lung segment would be the most problematic for this patient? 1) anterior basal segment of left lower lobe 2) apical segment of the upper lobe 3) superior segment of right lower lobe 4) posterior segment of right upper lobe

1) anterior basal segment of left lower lobe

May 17, 2020 A physical therapist treats a nine-year-old with osteogenesis imperfecta. Based on the diagnosis, which of the following would be most appropriate to include in the plan of care? 1) aquatic therapy exercises and instruction in the use of an AD 2) light resistance training and flexibility exercises 3) ambulation on a treadmill and cycling with resistance 4) balance training and coordination exercises

1) aquatic therapy exercises and instruction in the use of an AD

March 12, 2021 The medical record of a patient diagnosed with chronic obstructive pulmonary disease indicates that the patient's current condition is consistent with chronic respiratory acidosis. Which testing procedure was most likely used to diagnose respiratory acidosis? 1) arterial blood gas analysis 2) pulmonary function testing 3) graded exercise testing 4) pulse oximetry

1) arterial blood gas analysis

August 5, 2020 Recent diagnostic imaging identified the presence of a glenoid labral tear. Which diagnostic imaging method would most likely been used to identify this condition? 1) arthrogram 2) x-ray 3) doppler ultrasound 4) bone scan

1) arthrogram

March 24, 2020 A physical therapist treats a patient following R TKA in their home. The patient's bedroom is on the 2nd floor and the staircase has a handrail. Which functional task would likely be the first to occur during the rehab process? 1) ascending stairs w/ handrail and reciprocal pattern 2) ascending stairs w/o handrail and reciprocal pattern 3) descending stairs w/ handrail and reciprocal pattern 4) descending stairs w/o handrail and reciprocal pattern

1) ascending stairs w/ handrail and reciprocal pattern

March 13, 2020 An older adult being treated for general debility is found to be irritable, lethargic, and mildly confused. Lab testing reveals a vitamin B deficiency. This scenario is most consistent with: 1) atrophic gastritis 2) scurvy 3) dementia 4) osteomalacia

1) atrophic gastritis

March 6, 2020 A physician suspects a stress fracture in 34-year-old distance runner after completing an exam. Assuming the physician's preliminary diagnosis is correct, which of the following diagnostic tests would be the most appropriate to identify the stress fracture? 1) bone scan 2) fluoroscopy 3) telethermography 4) ultrasound scan

1) bone scan

April 25, 2020 A patient two days status post Cesarean delivery complains of incisional pain with coughing and sneezing. What is the most appropriate initial physical therapy intervention? 1) brace the incision with a pillow when coughing or sneezing 2) contract pelvic floor muscles 3) perform desensitization over the incision 4) initiate postural awareness activities

1) brace the incision with a pillow when coughing or sneezing

December 11, 2020 A physical therapist utilizes the should abduction test with a patient in sitting. The patient actively abducts their arm until the hand is resting on top of their head. The test position decreases the patient's symptoms. Based on this finding, which of the following conditions should the therapist most suspect? 1) cervical radiculopathy 2) TOS 3) impingement 4) subacromial bursitis

1) cervical radiculopathy

February 4, 2021 A physical therapist utilizes the shoulder abduction test with the patient in the sitting position and the patient's hand resting on the top of their head. The test position decreases the patient's symptoms. Based on this finding, which of the following should the therapist most suspect? 1) cervical radiculopathy 2) thoracic outlet syndrome 3) impingement syndrome 4) subacromial bursitis

1) cervical radiculopathy

July 11, 2020 A physical therapist determines that a patient is ready to progress to plyometric exercise. The patient reports starting a new medication since the last visit. Which of the following medications would be the most likely reason for the therapist deciding to postpone this exercise program? 1) ciprofloxacin (cipro) 2) celecoxib (celebrex) 3) atenolol (tenormin) 4) loratadine (claritin)

1) ciprofloxacin (cipro)

May 1, 2020 A physical therapist reviews the medical record of a patient who sustained a spinal cord injury. A note recently entered by the physician indicates that the patient contracted a respiratory infection. Which type of spinal cord injury would be the most susceptible to this condition? 1) complete C4 tetraplegia 2) cauda equina lesion 3) brown-sequard's syndrome 4) posterior cord syndrome

1) complete C4 tetraplegia

June 25, 2020 A patient rehabilitating from a spinal cord injury informs a therapist that he will walk again. Which type of injury would make function ambulation the least realistic? 1) complete T9 paraplegia 2) posterior cord snydrome 3) Brown-Sequard's syndrome 4) cauda equina injury

1) complete T9 paraplegia

January 23, 2021 A patient who has a spinal cord injury develops a respiratory infection. Which of the following diagnoses would make the patient the most susceptible to respiratory problems? 1) complete c4 tetraplegia 2) cauda equina lesion 3) Brown-Sequard syndrome 4) posterior cord syndrome

1) complete c4 tetraplegia

August 2, 2020 A physical therapist examines a 6 month old with Down Syndrome. Which of the following medical problems is most likely to further delay gross motor development in this population? 1) congenital heart failure 2) seizure disorder 3) atlantoaxial instability 4) obesity

1) congenital heart failure

April 20, 2020 A patient is examined in physical therapy for complaints of right elbow pain. When asked to indicate the location of their pain, the patient points to the anterior and lateral elbow. The therapist decides to perform a special test to differentiate between lateral epicondylitis and distal biceps tendonitis. Which of the following special tests would be the most useful? 1) cozens 2) pinch grip 3) elbow flexion test 4) bunnel-littler

1) cozens

February 17, 2021 A physical therapist reads in the medical record that a patient has recently been prescribed nitrates due to angina. What is the primary action of this pharmacological agent? 1) decrease cardiac preload and decrease cardiac afterload 2) decrease cardiac preload and increase cardiac afterload 3) increase cardiac preload and decrease cardiac afterload 4) increase cardiac preload and increase cardiac afterload

1) decrease cardiac preload and decrease cardiac afterload

Nov 6, 2019 A physical therapist reviews the role of the abdominal muscles during respiration. Which statement best describes the primary function of the internal oblique muscles during the process? 1) decrease the volume of the chest cavity during exhalation 2) decrease the volume of the chest cavity during inhalation 3) increase the volume of the chest cavity during exhalation 4) increase the volume of the chest cavity during inhalation

1) decrease the volume of the chest cavity during exhalation

June 13, 2020 A patient with severe scoliosis and complaints of dyspnea is treated in physical therapy for instruction in breathing exercises. The patient previously had spirometry testing to determine the extent of their respiratory dysfunction. Which of the following lung volumes would most likely be expected given the diagnosis? 1) decreased IRV 2) increased TV 3) decreased RV 4) increased ERV

1) decreased IRV

Dec 10, 2019 A physical therapist treats a patient diagnosed with neuropraxia of the ulnar nerve. Which of the following results should the therapist most expect from a nerve conduction velocity test on a patient with neuropraxia? 1) decreased nerve conduction velocity along the site of the lesion 2) decreased nerve conduction velocity only at points distal to the lesion 3) complete loss of electrical conduction at all points distal to the lesion 4) decreased nerve conduction velocity only at points proximal to the lesion

1) decreased nerve conduction velocity along the site of the lesion

Nov 20, 2019 An athlete sustains a grade II syndesmotic sprain of the ankle while playing football. Which of the following conditions is most likely to occur if this type of sprain is misdiagnosed or does not receive appropriate treatment? 1) degenerative articular cartilage changes 2) metatarsal stress fracture 3) anterior compartment syndrome 4) myositis ossificans

1) degenerative articular cartilage changes

June 26, 2020 The medical record indicates that a patient being treated for a low back strain is taking Lasix for the management of heart failure. During ambulation activities, the patient complains of lightheadedness, fatigue, and weakness. The therapist measures the patient's vital signs and identifies an irregular pulse. What is the most likely explanation? 1) dehydration 2) angina 3) heart failure 4) renal failure

1) dehydration

Dec 13, 2019 A patient diagnosed with lateral epicondylitis is referred to physical therapy. The therapist elects to use iontophoresis over the lateral epicondyle. Which type of current would the physical therapist use to administer the treatment? 1) direct 2) alternating 3) pulsatile 4) interferential

1) direct

August 6, 2020 A physical therapist performs goniometric measurements for elbow flexion with a patient in supine. In order to isolate elbow flexion, which structure should the therapist stabilize? 1) distal end of the humerus 2) proximal end of the humerus 3) distal end of ulna 4) proximal end of the radius

1) distal end of the humerus

Oct 26, 2019 A physical therapist performs goniometric measurements for elbow with a patient in supine. In order to isolate elbow flexion the therapist should stabilize the: 1) distal end of the humerus 2) proximal end of the humerus 3) distal end of the ulna 4) proximal end of the ulna

1) distal end of the humerus

January 29, 2021 A physical therapist works with a patient with a C7 tetraplegia on transferring from a manual wheelchair to a mat table with a sliding board. According to the head-hips rule, which direction should the patient move their head to initiate the transfer? 1) down and away from the mat 2) up and towards the mat 3) down and towards the mat 4) up and away from the mat

1) down and away from the mat

November 28, 2020 A patient appears to be somewhat anxious after learning her treatment will include soft tissue massage. What is the most appropriate massage stroke to begin treatment? 1) effleurage 2) vibration 3) petrissage 4) tapotement

1) effleurage

March 26, 2020 A patient who recently underwent diagnostic testing was prescribed Tegretol (carbamazepine) by their physician. Which diagnostic tool would most likely have resulted in the prescribing of this medication? 1) electroencephalography 2) lumbar puncture 3) cerebral angiography 4) echocardiography

1) electroencephalography

May 9, 2020 A physical therapist is concerned that a patient is developing signs and symptoms of rhabdomyolysis following an electrical burn. Upon reporting this information to the referring physician, blood tests are ordered. Which of the following findings would best support the therapist's hypothesis? 1) elevated myoglobin 2) hypokalemia 3) low creatine kinase 4) elevated pH

1) elevated myoglobin

December 17, 2020 A patient diagnosed with peripheral vascular disease begins a supervised treadmill exercise regimen intended to reduce claudication symptoms. After two minutes of ambulation, the patient reports the onset of claudication symptoms rated as 3/10 pain. What is the most appropriate response? 1) encourage the patient to continue until symptoms become more severe 2) reduce the speed and grade of the treadmill until symptoms resolve 3) allow the patient to rest, resuming ambulation once symptoms subside 4) discontinue the treadmill and select an alternative mode of exercise

1) encourage the patient to continue until symptoms become more severe

February 13, 2021 A physical therapist discusses modifiable risk factors for developing osteoporosis with a patient diagnosed with osteopenia. Which of the following factors would place the patient at the highest risk of developing osteoporosis? 1) excessive alcohol consumption 2) excessive intake of biphosphonates 3) inadequate intake of vitamin B12 4) inadequate intake of zinc

1) excessive alcohol consumption

March 19, 2020 A patient performs the exercise (SLR w/o full ext) following a TKA. Assuming the patient has full passive knee extension ROM, which term best describes what is occurring? 1) extension lag 2) disuse atrophy 3) flexion contracture 4) inhibition by pain

1) extension lag

November 18, 2020 A patient diagnosed with schizophrenia has been taking haloperidol (Haldol) following an acute psychotic episode. The physical therapist should be alert for which of the following side effects most commonly associated with this medication? 1) facial grimacing and involuntary extremity movements 2) significant weight gain and elevated serum lipids 3) constipation and urinary retention 4) respiratory depression and orthostatic hypotension

1) facial grimacing and involuntary extremity movements

Jan 21, 2020 A physical therapist works with a patient on the post-partum unit of an acute care hospital. The patient has a pubic symphysis separation. Which of the following interventions would be the most appropriate prior to discharge? 1) gait training with AD 2) abdominal strengthening 3) lumbar stabilization 4) progressive resistive exercises with elastic tubing

1) gait training with AD

August 30, 2020 A physical therapist examines an athlete during a football game and is concerned that the patient is experiencing heat exhaustion. Which of the following actions would be the least appropriate? 1) give the patient salt tablets to ingest 2) place the patient in supine with the legs elevated 3) have the patient drink fluids with electrolytes 4) loosen the patient's clothing

1) give the patient salt tablets to ingest

September 10, 2020 A physical therapist treats a patient with limited shoulder range of motion. The therapist hypothesizes that the patient's range of motion limitation is due to pain and not a specific tissue restriction. Which graded oscillation techniques would be the most appropriate to treat this patient? 1) grades 1,2 2) grades 2,3 3) grades 3,4 4) grades 4,5

1) grades 1,2

April 3, 2021 A physical therapist observes a patient's toes on the right foot often catch the ground during the swing phase of gait. Which objective finding would be most likely to contribute to this gait deviation? 1) hamstrings strength- fair minus 2) PF ROM 10 degrees 3) leg length discrepancy 1/2 inch 4) MAS: 2/4 for ant tib

1) hamstrings strength- fair minus

June 23, 2020 A physical therapist educates a patient status post transfemoral amputation on the importance of frequent skin checks. What is the most appropriate for the patient to utilize when inspecting the posterior aspect of the residual limb? 1) hand mirror 2) video camera 3) nurse 4) prosthetist

1) hand mirror

Jan 22, 2020 A patient rehabilitating from cardiac surgery receives orders for patient-controlled analgesia. Which of the following medications would most likely be administered with this method? 1) hydromorphone (dilaudid) 2) prednisone (deltasone) 3) digoxin (lanoxin) 4) haloperidol (haldol)

1) hydromorphone (dilaudid)

October 24, 2020 A patient with mallet finger admits to being noncompliant with the recommended splinting regimen. The patient was instructed to wear the prescribed splint continuously for 6 weeks. Which impairment would be most likely based on the described scenario? 1) inability to fully extend the DIP joint 2) inability to fully flex the DIP joint 3) inability to full extend the PIP joint 4) inability to fully flex the PIP joint

1) inability to fully extend the DIP joint

March 6, 2021 A physical therapist attempts to slow the progression of scoliosis and maintain respiratory function in a child with Duchenne muscular dystrophy by having the patient stand and ambulate with the spine in a slightly extended position. Which of the following results from maintaining the spine in slight extension? 1) increased WB through the facet joints 2) increased WB through the SI joint 3) increased WB through the anterior vertebral body 4) increased WB through the transverse processes

1) increased WB through the facet joints

A patient with a R CVA demonstrates pusher syndrome, which is most likely? 1) increased lean to L, increased L WB 2) increased lean to R, increased R WB 3) increased WB through R, unresponsive to stimuli on L 4) unequal WB and head rotated to L, unresponsive to stimuli on R

1) increased lean to L, increased L WB

September 24, 2020 A patient sustained an injury to the posterior cord of the brachial plexus post MVA. What muscle would not be affected by the injury? 1) infraspinatus 2) subscapularis 3) lats 4) teres major

1) infraspinatus

May 14, 2020 A physical therapist asks a patient several questions prior to administering cervical mechanical traction. During the questioning the patient indicates he is wearing dentures. What is the most appropriate therapist action? 1) initiate traction using normal treatment parameters with the dentures in place 2) initiate traction using normal treatment parameters after removing the dentures 3) initiate traction using 50% of the normal recommended force with the dentures in place 4) avoid using the traction since the intervention is contraindicated for the patient

1) initiate traction using normal treatment parameters with the dentures in place

October 1, 2020 A seven month old infant is able to initiate rolling from supine to prone when playing on a level floor surface. Which scenario would most likely be associated with this developmental milestone? 1) integration of the tonic labyrinthine reflex 2) persistence of the asymmetrical tonic reflex 3) integration of the rooting reflex 4) persistence of the galant reflex

1) integration of the tonic labyrinthine reflex

August 1, 2020 A physical therapist prepares to administer the Thessaly test to a patient with suspected meniscal tear in the right knee. Which activity would occur first when administering this test? 1) left extremity with knee in 20 degrees of flexion 2) left extremity with knee in 45 degrees of flexion 3) right extremity with knee in 20 degrees of flexion 4) right extremity with knee in 45 degrees of flexion

1) left extremity with knee in 20 degrees of flexion

Nov 11, 2019 A physical therapist works with a patient with hemiparesis. The patient is stabilizing the upper trunk and shifts weight onto the right ischium through: 1) left-sided lower body initiated lateral weight shift 2) right-sided lower body initiated lateral weight shift 3) lower body initiated anterior weight shift 4) lower body initiated posterior weight shift

1) left-sided lower body initiated lateral weight shift

July 10, 2020 A physical therapist instructs a patient to sit up slowly and wait until any dizziness passes prior to standing. A side effect of which of the following medications is the most likely reason for this precaution? 1) lisinopril (prinivil) 2) rosuvastatin (crestor) 3) enoxaparin (lovenox) 4) gabapentin (neurontin)

1) lisinopril (prinivil)

January 30, 2021 A physical therapist intends to use compression therapy as part of a patient's plan of care. The use of compression therapy would be contraindicated in which of the following impairments? 1) lower extremity edema due to CHF 2) decreased mobility following TKA 3) hypertrophic scarring following a burn to the LEs 4) residual limb edema following transfemoral amputation

1) lower extremity edema due to CHF

July 6, 2020 A physical therapist is considering the use of electrical stimulation on a chronic wound. Which of the following set of parameters is most appropriate? 1) monophasic, direct current 2) biphasic, direct current 3) monophasic, alternating current 4) biphasic, alternating current

1) monophasic, direct current

February 28, 2021 A physical therapist treats a patient who has multiple myeloma. Which of the following systems should the therapist expect to be targeted once symtpoms of the disease begin? 1) msk 2) cardiovascular 3) integumentary 4) neurologic

1) msk

March 21, 2021 A physical therapist treats a patient with hypoparathyroidism. Which of the following symptoms would most likely be associated with the patient's medical condition? 1) muscle cramps 2) weight loss 3) heat intolerance 4) moon-shaped face

1) muscle cramps

October 20, 2020 A physical therapist treats a patient with hypoparathyroidism. Which of the following symptoms would most likely be associated with the patient's medical condition? 1) muscle cramps 2) weight loss 3) heat intolerance 4) moon-shaped face

1) muscle cramps

September 1, 2020 A physical therapist employed in an acute care hospital routinely works with a pediatric population. Which of the following diagnoses would be most likely to have a latex allergy? 1) myelomeningocele 2) CP 3) down syndrome 4) MD

1) myelomeningocele

June 3, 2020 A female patient is an acute care hospital has a surgical procedure that results in an incision in the lower right abdominal area. The procedure was performed after an acute onset of the patient's condition. Which of the following symptoms most likely preceded the surgical procedure? 1) nausea and vomiting 2) bloody stools 3) flank pain 4) pain referred to right shoulder

1) nausea and vomiting

April 12, 2020 A physical therapist reviews a patient's x-ray with the patient's physician. The patient has osteoporosis and fractured their femur after a fall. Which of the following locations would be the most likely site for the fracture? 1) neck of femur 2) inter trochanteric space 3) linea alba 4) shaft of femur

1) neck of femur

March 4, 2021 A patient with HIV is being treated in PT for general deconditioning. What is the associated condition most likely to increase the risk for opportunistic infection? 1) neutropenia 2) anemia 3) polycythemia 4) thrombocytopenia

1) neutropenia

October 14, 2020 A physical therapist asks a patient to complete a visual analog scale designed to assess pain intensity. The scale consists of a 10 cm line with descriptive labels at each end. Which terminology would be the most appropriate for the first label? 1) no pain 2) mild pain 3) weak pain 4) faint pain

1) no pain

April 9, 2020 A patient is evaluated in physical therapy one week following a lateral ankle reconstruction. Which of the following weight bearing limitations would be most appropriate for this patient? 1) non-weight bearing w/ bilateral axillary crutches 2) partial weight bearing with a single axillary crutches 3) weight bearing as tolerated with bilateral axillary crutches 4) weight bearing as tolerated with a single axillary crutch

1) non-weight bearing w/ bilateral axillary crutches

December 4, 2020 A physical therapist treats a patient with the visual deficit strabismus. Which management technique would be the most effective for this particular deficit? 1) oculomotor and binocular exercises using a prism 2) convergence exercises to decrease diplopia 3) use of a headlamp to improve visual localization 4) scanning training to facilitate compensation

1) oculomotor and binocular exercises using a prism

Nov 8, 2019 A patient has been taking prednisone for nine months due to a chronic pulmonary disease. Which of the following conditions should the physical therapist be most concerned about as a potential risk factor for this patient? 1) osteoporosis 2) osteogenesis imperfecta 3) Type 1 diabetes 4) hyperthyroidism

1) osteoporosis

September 5, 2020 A physical therapist attempts to measure MCP abduction using a gonimeter. When conducting the measurement, what is the most appropriate location for the movable arm? 1) over the dorsal midline of proximal phalanx 2) over volar midline of the proximal phalanx 3) over the dorsal midline of the distal phalanx 4) over the volar midline of the distal phalanx

1) over the dorsal midline of proximal phalanx

September 4, 2020 A physical therapist treats a patient with a thermal agent that transfers heat using conduction. Which modality is consistent with this scenario? 1) paraffin 2) fluidotherapy 3) infrared 4) vapocoolant spray

1) paraffin

August 14, 2020 A patient rehabilitating form a fractured acetabulum is referred to physical therapy for ambulation activities. The patient has been on bed rest for 3 weeks and appears to be somewhat apprehensive about weight bearing. Which device would be the most appropriate to use when initiating ambulation activities? 1) parallel bars 2) walker 3) axillary crutches 4) straight cane

1) parallel bars

September 23, 2020 A patient demonstrates diminished problem-solving abilities and difficulty finding words at times. Additionally, they can only speak in low voice unless trying to yell, and appear to have a flattened affect.What medical condition is most consistent with the described presentation? 1) parkinson's 2) depression 3) MS 4) dementia

1) parkinson's

November 2, 2020 A physical therapist treats a patient diagnosed with Alzheimer's disease. Which of the following would be recommended as a primary intervention for this patient population? 1) physical activity 2) stretching 3) coordination training 4) pulmonary therapy

1) physical activity

September 9, 2020 A physical therapist considers an ankle-foot orthosis for a 62-year-old female who has difficulty with toe clearance during ambulation due to dorsiflexion weakness. Which objective finding would make a posterior leaf spring orthosis an inappropriate orthotic selection? 1) poor plantar strength 2) mild spasticity 3) good knee stability and strength 4) reduced proprioception at the ankle

1) poor plantar strength

September 11, 2020 A physical therapist working in a high school setting has an athlete come into the treatment room complaining of a headache, fever, painful neck. The therapist positions the athlete in supine and flexes the hip to 90 degrees. When the therapist attempts to extend the knee in this position, the patient complains of significant pain. What is this most indicative of? 1) positive Kernig's sign 2) negative Kernig's sign 3) positive Brudzinski's sign 4) negative Brudzinski's sign

1) positive Kernig's sign

Jan 7, 2020 A physical therapist works on transfer training with a patient who is currently taking a calcium channel blocker medication. Which side effect is most likely to occur during the session secondary to the use of this medication? 1) postural hypotension 2) tachycardia 3) sedation 4) cardiac arrythmias

1) postural hypotension

April 6, 2021 Following an acute MI, a patient continues to experience frquent arrhythmias. A PT should recognize an abnormality in which of the following lab values as being the most likely to contribute to the arrythmias? 1) potassium 2) hemoglobin 3) hematocrit 4) platelet count

1) potassium

Dec 31, 2019 A physical therapist performs a chart review for a patient admitted to the hospital after sustaining an acute myocardial infarction. A recent note in the medical record indicates that the patient has experienced frequent arrhythmias. An excessive amount of which nutrient would most likely contribute to the arrhythmias. 1) potassium 2) chloride 3) zinc 4) sodium

1) potassium

February 21, 2021 A patient diagnosed with CVA involving the anterior cerebral artery is referred to PT. Based on this diagnoses, which of the following interventions should be the physical therapist most anticipate to incorporate into the plan of care? 1) progression of LE strength and gait 2) facilitation techniques for weakness of UE and facial muscles 3) coordination/balance and referral to speech therapy 4) inhibitory techniques for UE spasticity and synergy patterns

1) progression of LE strength and gait

Feb 21, 2020 A patient diagnosed with a CVA involving the anterior cerebral artery is referred to physical therapy. Based on the diagnosis, which of the following interventions should the physical therapist most anticipate needed to incorporate into the plan of care for this patient? 1) progression of lower extremity strengthening exercises and gait activities 2) facilitation techniques for weakness of upper extremity and facial muscles 3) coordination/balance activities and referral to speech therapy for aphasia 4) inhibitory techniques for upper extremity spasticity and synergy patterns

1) progression of lower extremity strengthening exercises and gait activities

November 12, 2020 A patient demonstrates difficulty ambulating due to bilateral pain in the knees. The patient reports onset of pain was sudden and seemed to occur after a recent episode of the flu. Radiography reveals evidence of calcium deposits in the knee joints. What is this scenario most consistent with? 1) pseudogout 2) gout 3) polymyalgia rhumatica 4) systemic lupus erythematosus

1) pseudogout

December 28, 2020 A physical therapist interviews a patient referred to physical therapy with a diagnosis of chronic bronchitis. What finding obtained during the patient interview is most closely associated with the patient's medical diagnosis? 1) pt is a smoker 2) pt is sedentary 3) pt has numerous allergies 4) pt has severe scoliosis

1) pt is a smoker

April 29, 2020 A physical therapist implements an exercise program for a patient that is 27 weeks pregnant. Which position would be the most desirable for exercise activities based on the patient's current status? 1) quadruped and sitting 2) supine and hooklying 3) sitting and trendelenburg 4) trendelenburg and quadruped

1) quadruped and sitting

Jan 29, 2020 A patient with cardiac arrhythmias has been prescribed beta-blockers. What is the most effective way to monitor the patient's activity tolerance? 1) rating of perceived exertion 2) visual analog scale 3) palpation of radial heart rate 4) assessment of telemetry strip

1) rating of perceived exertion

May 3, 2020 A physical therapist treating a patient who has RA identifies the presence of hallux valgus. The patient reports pain and tenderness in the area of the great toe. Which of the following actions would be the most beneficial to address this condition? 1) recommend a shoe with a wide toe box 2) improve alignment of the great toe by using a metatarsal bar 3) increase flexion ROM of the great toe 4) use a heel cup to redistribute forces

1) recommend a shoe with a wide toe box

Feb 11, 2020 A patient post stroke presents with significant impairments in kinesthesia during sensory testing of the involved extremities. Based on this finding, which of the following proprioceptive neuromuscular facilitation techniques would be the most appropriate intervention? 1) rhythmic initiation 2) alternating isometrics 3) rhythmic stabilization 4) hold-relax

1) rhythmic initiation

July 27, 2020 A physical therapist estimates that a patient has lost 20% of their strength in their right hip adductors after sustaining a groin injury two months ago. Which position would be the most appropriate when performing a manual muscle test of the right hip adductors based on the hypothesized strength deficit? 1) right sidelying 2) left sidelying 3) prone 4) supine

1) right sidelying

August 18, 2020 A patient with DM is scheduled to walk approximately one-half mile as part of a community fitness initiative. The therapist checks the patient's blood glucose before initiating activity and finds it is 100 mg/dL. This finding indicates that the therapist should make which of the following recommendations? 1) safe to perform the activity 2) patient should increase food intake with fruit or bread 3) patient should increase food intake with half sandwich with fruit or milk 4) patient should not exercise

1) safe to perform the activity

Nov 29, 2019 A physical therapist treats a patient that presents with a rapid and shallow breathing pattern as well as external dyspnea. Pulmonary function testing reveals decreased vital capacity and total lung capacity, however, residual volume and expiratory flow rates are within normal limits. Which of the following conditions is the most likely extrapulmonary etiology with these clinical findings? 1) scoliosis 2) sarcoidosis 3) tuberculosis 4) chronic bronchitis

1) scoliosis

January 6, 2021 A physical therapist reviews a patient's medical record prior to beginning treatment. The record indicates the patients was recently place on amitripyline (Elavil). Which of the following responses is the most common side effect associated with this antidepressant? 1) sedation 2) dysartria 3) seizures 4) blood pressure variability

1) sedation

March 5, 2020 A patient with osteoarthritis receives a whirlpool treatment using a Hubbard tank to decrease muscle and joint stiffness. The patient's medications include a calcium channel blocker for hypertension. The most relevant potential interaction between for calcium channel blocker and described intervention is: 1) severe hypotension 2) local vasoconstriction 3) neuromuscular junction stimulation 4) cutaneous hypersensitivity

1) severe hypotension

October 4, 2020 A patient develops lymphedema within her right upper extremity following treatment for breast cancer. The patient is directed to use compression bandages to help control fluctuations in edema levels. Which of the following bandages would be most appropriate for this patient? 1) short-stretch, more compression distally 2) short-stretch, more compression proximally 3) long-stretch, more compression distally 4) long-stretch, more compression proximally

1) short-stretch, more compression distally

October 11, 2020 A 10-year-old child with spina bifida has recurring headaches, fever, and decreased activity level. What should the physical therapist suspect? 1) shunt dysfunction 2) flu 3) latex allergy 4) seizures

1) shunt dysfunction

May 20, 2020 A physical therapist observes what appears to be an alkaline reaction after administering several treatments of iontophoresis. Which finding would be most anticipated as a result of the described reaction? 1) sodium hydroxide at the cathode 2) sodium hydroxide at the anode 3) hydrochloric acid at the cathode 4) hydrochloric acid at the anode

1) sodium hydroxide at the cathode

November 27, 2020 A physical therapist inspects a burn obtained as a result of iontophoresis. The therapist describes the burn as an alkaline reaction. Which of the following factors would most likely be the cause of this type of adverse reaction? 1) sodium hydroxide forming under the cathode 2) hydrochloric acid forming under the anode 3) size of the cathode being larger than the anode 4) an increase in the amount of space between the electrodes

1) sodium hydroxide forming under the cathode

February 11, 2021 A physical therapist prepares to initiate an exercise program for a patient whose current medical history includes acid reflux disease. Which activity would likely be the most problematic for the patient? 1) strengthening using cuff weights in semi-fowlers position 2) walking 4-5 mph on a treadmill 3) resistive exercise using an upper body ergometer 4) squatting exercise using a resistance band

1) strengthening using cuff weights in semi-fowlers position

Dec 2, 2019 A physical therapist evaluating an infant observes the patient manipulating objects. Which of the following manipulation skills demonstrate the most advanced volitional movement? 1) striking two blocks together 2) holding an object in one hand and banging it with the other 3) transferring an object between hands 4) shaking of held object in either hand

1) striking two blocks together

March 17, 2020 A physical therapist instructs a patient in an exercise designed to increase pelvic floor awareness and strength. The exercise requires the patient to tighten the pelvic floor as if attempting to stop the floor of urine. The patient is instructed to hold to isometric contraction for 5 seconds and complete 10 repetitions. The most appropriate initial position for this exercise would be: 1) supine 2) sitting 3) tall kneeling 4) standing

1) supine

April 7, 2020 A patient is evaluated in physical therapy three weeks following a lateral ankle reconstruction. Which of the following exercises for the involved lower extremity would be the most appropriate to include as part of this patient's home exercises program on the first session? 1) supine straight leg raise 2) ankle inversion/eversion with a resistance band 3) standing calf raise with upper extremity support 4) single leg stance with eyes open

1) supine straight leg raise

Dec 19, 2019 A physical therapist examines the posture of a patient from a lateral view using a plumb line. Which structural condition would be most likely to cause the plumb line to fall further posterior to the hip joint? 1) swayback 2) lordosis 3) scoliosis 4) genu recurvatum

1) swayback

Feb 23, 2020 A 68-year-old female patient diagnosed with polymyalgia rheumatica reports recently experiencing headaches of increasing intensity along with visual disturbances. Based on this clinical presentation, which of the following conditions is the most likely cause of this patient's symptoms? 1) temporal arteritis 2) trigeminal neuralgia 3) myofascial pain syndrome 4) transient ischemic attack

1) temporal arteritis

Jan 31, 2020 An older adult patient presents with shoulder and neck pain secondary to polymyalgia rheumatica. They express that recently they have been experiencing headaches of increasing intensity along with visual disturbances. What is the most likely reason for this complaint? 1) temporal arteritis 2) migraine 3) cervicogenic headache 4) transient ischemic attack

1) temporal arteritis

January 27, 2021 A physical therapist attempts to assess the motor component of the axillary nerve by conducting a manual muscle test. Which of the following muscles would be the most appropriate to utilize? 1) teres minor 2) teres major 3) subscapularis 4) supraspinatus

1) teres minor

March 8 , 2021 The result of the jaw jerk reflex suggest that a patient may have an upper motor neuron lesion. Based on the testing procedure, which finding would best support the stated hypothesis? 1) the mandible moves upward 2) the mandible moves downward 3) the mandible moves laterally 4) the mandible remains in the testing position

1) the mandible moves upward

February 7, 2021 A physical therapist examines a patient recently diagnosed with adhesive capsulitis. Based on the patient's subjective reports and examination findings, the therapist believes the patient is in the acute phase of the condition. Which finding tends to be unique to the acute phase of adhesive capsulitis? 1) the patient's sleep is disrupted by nocturnal pain 2) ROM is limited by muscle weakness 3) pain complaints progressively decrease 4) the patient demonstrates a capsular pattern of restriction

1) the patient's sleep is disrupted by nocturnal pain

Feb 12, 2020 A physical therapist monitors an adult patient's blood pressure using the brachial artery, though they must use a pediatric blood pressure cuff since it is the only one available in the clinic. What effect would this have on the patient's blood pressure measurement? 1) the reading will be higher than the actual measurement 2) the reading will be lower than the actual measurement 3) the reading will be accurate if the therapist deflates the cuff at 3 mm Hg per second 4) the reading will be accurate if the therapist deflates the the cuff at 1 mm Hg per second

1) the reading will be higher than the actual measurement

Nov 17, 2019 A patient presents with marked weaknesses during resisted testing of right shoulder extension and medial (internal) rotation. Which of the following nerve lesions would most likely produce the described impairments? 1) thoracodorsal nerve 2) suprascapular nerve 3) spinal accessory nerve 4) axillary nerve

1) thoracodorsal nerve

January 21, 2021 A patient uses a fixed support strategy to control backward sway when in a standing position. Which muscle would most likely be activated first in this scenario? 1) tibialis anterior 2) gastroc 3) hamstrings 4) paraspinals

1) tibialis anterior

March 28, 2020 A patient with Alzheimer's disease residing in an assisted living facility exhibits sundown syndrome. Which of the following actions is the most appropriate to diminish the severity of sundowning symptoms? 1) use of increased lighting in the early evening 2) encourage naps during the daytime 3) decrease general activity level 4) reduce structure of patient's daily routine

1) use of increased lighting in the early evening

Nov 23, 2019 A patient with Alzheimer's disease residing in an assisted living facility exhibits sundown syndrome. Which of the following actions is the most appropriate to diminish the severity of sundowning symptoms? 1) use of increased lighting in the early morning 2) encourage naps during the day 3) decrease general activity level 4) reduce structure of patient's daily routine

1) use of increased lighting in the early morning

December 1, 2020 A physical therapist reviews the results of a pulmonary function test. Assuming normal values, which of the following measurements would you expect to be the greatest? 1) vital capacity 2) tidal volume 3) residual volume 4) inspiratory reserve volume

1) vital capacity

Feb 14, 2020 A physical therapist prepares to complete a manual muscle test of the opponens pollicis. Wheres should the therapist apply pressure for providing resistance when completing this muscle test? 1) volar surface of the distal end of the metacarpal 2) dorsal surface of the proximal phalanx of the first digit 3) volar surface of the distal phalanx of the first digit 4) volar surface of the trapezium and trapezoid

1) volar surface of the distal end of the metacarpal

July 22, 2020 A 29-year-old status post Colles' fracture is referred to physical therapy. The patient has moderate edema in her fingers and the dorsum of her hand and complains of pain during active range of motion. Which method is most appropriate to quantify the patient's edema? 1) volumetric measurements 2) circumferential measurements 3) girth measurements 4) anthropometric measurements

1) volumetric measurements

April 4, 2021 A PT determines that a patient's abnormal gait is characterized as ataxic. Which of the following best describes this gait pattern? 1) wide base of support with exaggerated movements 2) legs grossing the midline 3) small shuffling and festinating 4) high stepping with foot slap

1) wide base of support with exaggerated movements

Jan 2, 2020 A physical therapist determines that a patient's abnormal gait pattern is characterized as ataxic. Which of the following best describes this gait pattern? 1) wide base of support with exaggerated movements 2) legs crossing the midline 3) small shuffling steps and festinating 4) high stepping with foot slap

1) wide base of support with exaggerated movements

May 4, 2020 A physical therapist examines a patient who has lateral epicondylitis. Based on this diagnosis, the therapist should expect the patient to experience pain with muscle testing which of the following motions? 1) wrist extension and forearm supination 2) wrist extension and forearm pronation 3) wrist flexion and forearm supination 4) wrist flexion and forearm pronation

1) wrist extension and forearm supination

April 10, 2021 A pt is required to wear a thumb spica splint following a traumatic fx of the 1st metacarpal. Which of the following movements would the pt have the most difficulty performing while wearing the splint? 1) wrist flex 2) pronation 3) ext of digits 2-5 4) elbow flex

1) wrist flex

Dec 9, 2019 A physical therapist measures passive forearm supination and concludes that the results are within normal limits. Which measurement would be classified as within normal limits? 1) 0-60 2) 0-80 3) 0-100 4) 0-120

2) 0-80

October 31, 2020 A physical therapist employed in a rehabilitation hospital discusses the POC for a patient status post unilateral transtibial amputation. Assuming an uncomplicated recovery, what is the most appropriate amount of time for prosthetic training? 1) 1-2 days 2) 1-2 weeks 3) 4-6 weeks 4) 6-8 weeks

2) 1-2 weeks

Feb 10, 2020 A physical therapist conducts a goniometric assessment of a patient's glenohumeral joint to differentiate this motion from full motion of the shoulder complex. Which of the following values is most indicative of normal passive glenohumeral abduction? 1) 60 degrees 2) 120 degrees 3) 155 degrees 4) 180 degrees

2) 120 degrees

November 10, 2020 A physical therapist participates in a scoliosis screening as part of a health and wellness fair. Which of the individuals would be the best candidate to participate in the screening process? 1) 10 yo male 2) 13 yo male 3) 8 yo male 4) 16 yo female

2) 13 yo male

June 30, 2020 A physical therapist monitors the blood pressure of a patient walking on a treadmill at 3.0 MPH. Prior to initiating the exercise, the therapist recorded the patient's resting blood pressure as 136/81 mmHg. Which blood pressure value obtained during the exercise would be the most concerning? 1) 148/78 2) 158/93 3)167/77 4) 173/85

2) 158/93

Nov 16, 2019 A physical therapist administers superficial heat to the low back of a patient using a hydrocollator pack. Assuming the therapist uses 6 towel layers, the most appropriate duration for the intervention is: 1) 10 minutes 2) 20 minutes 3) 30 minutes 4) 40 minutes

2) 20 minutes

August 17, 2020 A physical therapist uses the body mass index scale as a means of assessing a patient's total body composition. Which of the following values would be the most representative of a healthy adult? 1) 14 2) 22 3) 28 4) 37

2) 22

December 20, 2020 A physical therapist prepares to treat a patient using an oxygen tent. Which scenario would best support this method of oxygen administration? 1) 54 male rehabbing from an MI 2) 3 female rehabbing from a head injury 3) 34 female with multi-trauma following a car accident 4) 47 make with COPD

2) 3 female rehabbing from a head injury

May 22, 2020 A physical therapist assess a patient with suspected unilateral hearing loss using the Rinne test. With the tuning fork on the patient's skull, the patient reports that the sound disappears after 15 seconds. Assuming the patient's sense of hearing is normal, how long should the patient hear the sound when the tuning fork is placed next to their ear? 1) 15 seconds 2) 30 seconds 3) 45 seconds 4) 60 seconds

2) 30 seconds

March 16, 2020 A patient in the hospital following surgery sets the target indicator on an incentive spirometer. Which individual would most likely have the highest target value when using the device? 1) 35 yo female, 66 inches tall, post transmyocardial revascularization 2) 45 yo male, 72 inches tall, post TKA 3) 55 yo female, 64 inches tall, post cardiac valve replacement 4) 55 yo male, 70 inches tall, post coronary artery bypass graft

2) 45 yo male, 72 inches tall, post TKA

August 19, 2020 A physical therapist assesses a patient's level of independence using the FIM. If the patient needs only supervision when bathing, what FIM score would be most appropriate? 1) 7 2) 5 3) 3 4) 1

2) 5

March 4, 2020 A physical therapist uses a floor scale to educate a patient on their current weight bearing status following a lower extremity injury. Assuming the patient weighs 250 pounds and has been cleared for 30% weight bearing, what would be the most appropriate amount of weight transmitted through the involved lower extremity? 1) 50 pounds 2) 75 pounds 3) 100 pounds 4) 175 pounds

2) 75 pounds

September 15, 2020 A patient using a wheelchair arranges for a local contractor to build a ramp that will allow entry into the patient's house. What is the maximum recommended grade for the ramp? 1) 6.2% 2) 8.3% 3) 9.5% 4) 10.4%

2) 8.3%

August 13, 2020 A physical therapist treats a patient that was diagnosed with ALS 18 months ago. The patient is experiencing increased difficulty performing activities of daily living and strength is grossly 3/5 for LE and 3+/5 for the UE. Which intervention would be the most appropriate to include in the patient's plan of care? 1) PROM 2) AROM 3) resistance exercise program 4) endurance training

2) AROM

Dec 14, 2019 During an examination of a patient with chronic shoulder pain, a physical therapist attempts to determine the patient's level of kinesiophobia. What objective information would be the most useful for the therapist? 1) reflex activity 2) AROM 3) PROM 4) sensory testing

2) AROM

February 15, 2021 A physical therapist treats a patient that was diagnosed with ALS 18 months ago. The patient is experiencing increased difficulty performing ADLs and their strength is grossly 3/5 for the lower extremities and the upper extremities. Which intervention would be the most appropriate to include in the patient's plan of care? 1) PROM 2) AROM 3) resistance exercise program 4) endurance training

2) AROM

August 23, 2020 A physical therapist recognizes that an increase in venous return during exercise stretches receptors in the wall of the right atrium which ultimately inhibits parasympathetic activity. This inhibition of parasympathetic activity results in an increased heart rate. 1) baroreceptor reflex 2) Bainbridge reflex 3) chemoreceptor reflex 4) Valsalva maneuver

2) Bainbridge reflex

Nov 30, 2019 A 12-year-old female diagnosed with adolescent idiopathic scoliosis presents with a right thoracic curve of 30 degrees. Which form of medical management would be most likely for this patient? 1) halo vest 2) Boston brace 3) bone growth stimulator 4) spinal fusion

2) Boston brace

Feb 2, 2020 A physical therapist reviews the chart of a patient diagnosed with a traumatic spinal cord injury. The patient's motor level was determined to be C7 and the sensory level was determined to be C8. The neurological level of the injury for this patient would be best described as: 1) C6 2) C7 3) C8 4) T1

2) C7

March 2, 2020 A physical therapist reviews the chart of a patient diagnosed with traumatic spinal cord injury. The patient's motor level was determined to be C7 and sensory level was determined to be C8. The neurological level of injury for this patient would be best described as: 1) C6 2) C7 3) C8 4) T1

2) C7

Jan 10, 2020 A patient with a genetically inherited neuromuscular disorder reports weakness in their hands and feet with normal strength elsewhere. The patient experiences difficulty with grasping objects and demonstrates a steppage gait pattern. This clinical presentation is most consistent with which of the following medical conditions? 1) Guillain-Barre syndrome 2) Charcot-Marie-Tooth disease 3) Post-polio syndrome 4) Huntington's disease

2) Charcot-Marie-Tooth disease

August 4, 2020 A female patient with osteopenia is placed on hormone replacement therapy by her physician. Hormone replacement therapy places the patient at greatest risk for which of the following conditions? 1) aneurysm 2) DVT 3) vertebral compression fx 4) colon cancer

2) DVT

Jan 3, 2020 A physician shows a physical therapist an x-ray taken on a child that depicts an osteochondroma. Which location would be the most likely for the osteochondroma? 1) Neck of the femur 2) Distal femur 3) Proximal radius 4) Distal humerus

2) Distal femur

June 9, 2020 A patient with idiopathic pulmonary fibrosis reviews the results of their spirometry testing with their physician. The physician explains the relevance of the FEV1 (forced expiratory volume in 1 sec) and FVC (forced vital capacity) values. Based on the diagnosis, which of the following values would most likely be expected? 1) FEV1= 3 L; FVC = 5 L 2) FEV1= 2 L; FVC= 2.5 L 3) FEV1= 3.5 L; FVC= 6 L 4) FEV1- 2.5; FVC= 4 L

2) FEV1= 2 L; FVC= 2.5 L

March 29, 2020 A physical therapist examines differences in lung volumes based on gender. Which lung volume would the therapist anticipate having the greatest difference between males and females? 1) FRC 2) IRV 3) ERV 4) RV

2) IRV

January 9, 2021 A physical therapist reporting at a team meeting indicates that a patient with a spinal cord injury should be able to perform household ambulation using KAFOs and forearm crutches upon discharge. The patient's quads are 3/5. What level of SCI is this based on therapist's predication? 1) L1 2) L3 3) L5 4) S1

2) L3

December 7, 2020 A physical therapist performs a valgus stress test on a patient who sustained a knee injury during a football game. The therapist determines that the test is positive with the knee in 30 degrees of flexion, but negative with the knee in full extension. These findings are most consistent with what knee injury? 1) MCL and PCL 2) MCL and posteromedial capsule 3) MCL and ACL 4) MCL and LCL

2) MCL and posteromedial capsule

Nov 13, 2019 A physical therapist treats a patients diagnosed with epilepsy that is taking an anticonvulsant medication. The physician hypothesizes that the medication may be responsible for the patient's recent cardiac arrhythmias. What anticonvulsant would be most likely to produce this type of side effect? 1) Dilantin 2) Tegretol 3) Klonopin 4) Neurontin

2) Tegretol

February 10, 2021 A physical therapist examines a patient with an acute burn to the left shoulder and axilla area secondary to a spill of boiling water. What is the most appropriate splint for this patient? 1) shoulder sling 2) airplane splint 3) c-bar splint 4) hemi-arm sling

2) airplane splint

Jan 4, 2020 A physical therapist assigns a grade of fair after performing a manual muscle test with a patient in sitting. Which muscle would most likely be associated with the described testing procedure? 1) pec major 2) anterior deltoid 3) latissimus dorsi 4) lower trap

2) anterior deltoid

August 16, 2020 A physical therapist reviews the medical record of a patient post MI. The patient is currently in the coronary care unit and is scheduled to begin cardiac rehab tomorrow. This patient is most susceptible to which of the following potential complications of a MI? 1) heart failure 2) arrhythmias 3) thrombus formation 4) heart structural damage

2) arrhythmias

March 7, 2020 A physical therapist walks with a patient in an inpatient cardiac rehabilitation unit. The patient describes a sensation of lower extremity cramping and sensory changes after a distance of 50 feet. The therapist hypothesizes that the patient's symptoms are consistent with pain from claudication. Which of the following tests would be the most appropriate to confirm the hypothesis? 1) venous doppler ultrasonography 2) arterial doppler ultrasonography 3) clinical electromyography 4) electrocardiogram

2) arterial doppler ultrasonography

December 10, 2020 A physical therapist examines a patient diagnosed with an ACL sprain. If the therapist wants to quantitatively measure the anteroposterior translation at the knee, which of the following instruments would provide the most accurate measurement? 1) goni 2) arthrometer 3) tape measure 4) dynamometer

2) arthrometer

Jan 8, 2020 A physical therapist is treating a five-year-old child with a diagnosis of athetoid cerebral palsy involving all four of their extremities. Which structure was most likely affected with this type of cerebral palsy? 1) motor cortex 2) basal ganglia 3) cerebellum 4) brainstem

2) basal ganglia

October 25, 2020 A patient with C5 tetraplegia exercises on a mat table. Suddenly, the patient begins to demonstrate signs and symptoms of autonomic dysreflexia including headache and sweating above the level of the lesion. What is the most appropriate assessment to validate the presence of autonomic dysreflexia? 1) pulse rate 2) blood pressure 3) respiratory rate 4) oxygen saturation

2) blood pressure

February 26, 2021 A physical therapist works with a 4 month old infant. During mat activities the infant suddenly becomes unconscious. Which of the following arteries is the most appropriate for the therapist to palpate to assess the infant's pulse? 1) radial 2) brachial 3) popliteal 4) carotid

2) brachial

May 6, 2020 A physical therapist treats a patient who has an entrance wound on the top of their head and an exit wound on the anterior right thigh secondary to coming into contact with a live power line while at work. Which of the following complications should the therapist be most alert for given the type of injury this patient sustained? 1) hypervolemia 2) cardiac arrhythmia 3) pulmonary fibrosis 4) heterotopic ossification

2) cardiac arrhythmia

Feb 15, 2020 A physical therapist treats a patient diagnosed with epilepsy who is taking carbmazapine (Tegretol). Which of the following potential adverse effects should the therapist recognize as a primary side effect of this medication? 1) muscle wasting or atrophy 2) cardiac arrhythmias 3) joint pain and swelling 4) respiratory depression

2) cardiac arrhythmias

June 1, 2020 A physical therapist instructs a patient to perform body blade shakes on a foam pad. What is the purpose of the foam pad? 1) add resistance 2) challenge balance 3) add stability 4) change the person's ankle flexibility

2) challenge balance

Nov 14, 2019 A physical therapist instructs a patient to perform SL stance on an airex. The primary purpose of using the airex is to: 1) add resistance to the exercise 2) challenge balance 3) add stability 4) challenge ankle flexibility

2) challenge balance

July 26, 2020 A patient rehabilitating from multi-trauma injuries sustained in a MVA is not a candidate for patient-controlled analgesia. Which finding may have been most relevant when making this determination? 1) surgical contraindications 2) cognitive impairments 3) functional limitations 4) GI distress

2) cognitive impairments

August 27, 2020 A patient is admitted to an acute care hospital following a motor vehicle accident. The patient is presently in a state of unconsciousness without arousal, eye-opening, or sleep-wake cycles. The patient does possess intact brainstem reflexes upon examination. What diagnosis is most likely? 1) brain death 2) coma 3) vegetative state 4) locked-in syndrome

2) coma

March 7, 2021 During a patient interview, a physical therapist discovers that a patient is two weeks post vertebroplasty. This procedure was most likely performed for which of the following medical conditions? 1) disk herniation 2) compression fracture 3) spinal instability 4) spondylosis

2) compression fracture

March 17, 2021 A physical therapist treats a patient with subjective complaints of aching in the medial forearm and ulnar side of the hand. The patient indicates that repetitive elbow flexion seems to exacerbate symptoms. Testing reveals a positive Tinel's sign. What is the patient's most likely diagnosis? 1) carpal tunnel 2) cubital tunnel 3) lateral epicondylitis 4) radial tunnel

2) cubital tunnel

Feb 4, 2020 A physical therapy plan of care to manage contractures in an adolescent with Duchenne muscular dystrophy should include: 1) general manual stretching of the extremities 2) daily standing program 3) contract/relax stretching strategies for tight muscles 4) progression of walking activities

2) daily standing program

January 16, 2021 A physical therapist documents gait training performed during a treatment session. Which descriptive term is only associated with the swing phase of gait cycle? 1) heel strike 2) deceleration 3) loading response 4) midstance

2) deceleration

April 2, 2020 A physical therapist applies kinesiology tape as depicted in the image to a patient's knee three weeks following knee surgery. What is the most likely therapist objective when applying the tape as shown in the image? (image shows tape cut into long strands and all spread out in a fan-like way) 1) decrease pain 2) decrease swelling 3) increase ROM 4) increase strength

2) decrease swelling

Dec 22, 2019 An 82-year-old patient reports fatigue when ambulating to and from their home mailbox. Which of the following factors would most likely contribute to the patient's reported difficulty? 1) increased diastolic blood pressure 2) decreased cardiac output 3) decreased cardiac afterload 4) decreased arterial patency

2) decreased cardiac output

November 11, 2020 A physical therapist measures the strength of the iliopsoas in sitting, however, after performing the test the therapist realizes that the hamstrings were not on slack. What observation would have been most likely during the testing? 1) increase hip flexion ROM 2) decreased hip flexion ROM 3) increased hip ext ROM 4) decreased hip ext ROM

2) decreased hip flexion ROM

Dec 28, 2019 A physical therapist examines the gait of a patient with peripheral neuropathy. The therapist observes that the patient's right foot tends to slap the ground during the loading response. Damage to which of the following nerves would best explain this gait impairment? 1) tibial 2) deep peroneal 3) superficial peroneal 4) femoral

2) deep peroneal

June 22, 2020 A note in a patient's medical record indicates that laboratory testing identified an elevated hematocrit. Which condition would most likely be associated with this finding? 1) anemia 2) dehydration 3) leukemia 4) nutritional deficiency

2) dehydration

September 8, 2020 While receiving physical therapy, a patient experiences a syncopal event. A review of the patient's medical record indicates that the patient has had both diarrhea and vomiting within the last 24 hours. Which of the following conditions is the most likely cause of this syncopal event? 1) anemia 2) dehydration 3) orthostatic hypotension 4) pregnancy

2) dehydration

October 15, 2020 A patient presents with LUE lymphedema post mastectomy. The physical therapist plans to initiate lymphatic drainage exercises as part of the POC. Which of the following breathing exercises is the most appropriate for the therapist to teach the patient to perform prior to initiating lymphatic drainage? 1) segmental breathing 2) diaphragmatic breathing 3) paced breathing 4) inspiratory muscle training

2) diaphragmatic breathing

Jan 9, 2020 A physical therapist positions a patient in a hooklying position and asks the patient to raise their head from the plinth. This testing procedure would most likely be used to identify which condition? 1) vertebral artery compression 2) diastasis recti 3) thoracic outlet syndrome 4) acromioclavicular arthritis

2) diastasis recti

Feb 16, 2020 A patient with heart failure has been prescribed digoxin (Lanoxin) as part of the pharmacological management of an exacerbation of the disease. Which of the following adverse signs or symptoms should the physical therapist be most alert for during exercise based on the prescribed medication? 1) bradycardia and hemorrhage 2) dizziness and weakness 3) headache and migraines 4) involuntary movements and tremors

2) dizziness and weakness

May 7, 2020 A patient presents with a flexion contracture of the elbow after sustaining deep partial-thickness and full-thickness burns to the antecubital region. Which of the following splints would be most appropriate to address this impairment? 1) static splint applied at the end range of elbow extension with forearm pronation 2) dynamic splint with low-load stress into elbow extension with forearm supination 3) static splint applied at the end range of elbow flexion with the forearm in a neutral position 4) dynamic splint with low-load stress into elbow flexion with forearm supination

2) dynamic splint with low-load stress into elbow extension with forearm supination

Feb 28, 2020 A physical therapist assesses the functional strength of a patient's hip extensors while observing the patient move from standing to sitting. What type of contraction occurs in the hip extensors during this activity? 1) concentric 2) eccentric 3) isometric 4) isokinetic

2) eccentric

Nov 2, 2019 A patient diagnosed with patellofemoral syndrome reports significant pain in the involved knee when descending stairs. Which scenario is most likely responsible for the patients subjective report of knee pain? 1) concentric activity of the quadriceps 2) eccentric activity of the quadriceps 3) concentric activity of the hamstrings 4) eccentric activity of the hamstrings

2) eccentric activity of the quadriceps

April 11, 2020 A patient is referred to physical therapy with a diagnosis of lateral epicondylitis. During the examination, the therapist attempts to elicit the patient's pain by placing the affected tissues on a maximal stretch. Which combination of motions would most likely achieve the desired effect? 1) elbow extension, forearm supination, wrist extension 2) elbow extension, forearm supination, wrist flexion 3) elbow flexion, forearm pronation, wrist extension 4) elbow flexion, forearm supination, wrist flexion

2) elbow extension, forearm supination, wrist flexion

Dec 8, 2019 A physical therapist reviews the medical record of a patient diagnosed with Guillain-Barre syndrome. A recent lumbar puncture helped confirm the diagnosis. Which of the following findings would most likely have been present in the cerebrospinal fluid sample to confirm the medical diagnosis? 1) elevated levels of erythrocytes 2) elevated protein levels 3) low levels of gamma globulin 4) low glucose levels

2) elevated protein levels

July 1, 2020 A physical therapist administers a coupling agent to the forearm of a patient prior to performing an ultrasound treatment. What is the primary purpose of the coupling agent? 1) promote patient comfort during the session 2) eliminate air between the transducer and the skin 3) enhance the depth of penetration 4) minimize the likelihood of skin irritation

2) eliminate air between the transducer and the skin

October 3, 2020 A physical therapist utilizes a manual assisted cough technique on a patient who has a mid-thoracic spinal cord injury. When completing this technique with the patient in the supine position, which of the following locations is the most appropriate for placement of the therapist's hands? 1) manubrium 2) epigastric area 3) xiphoid process 4) umbilical region

2) epigastric area

December 18, 2020 A physical therapist instructs a patient status post abdominal surgery how to use an incentive spirometer in order to prevent the occurrence of pulmonary complications. What is the minimum frequency for the patient to use the incentive spirometer when they are awake? 1) every 5-10 min 2) every 1-2 hours 3) twice a day 4) once a day

2) every 1-2 hours

Jan 26, 2020 A patient 72 hours status post stroke is referred to physical therapy. As part of the patient care program, the physical therapist makes positioning recommendations to the nursing staff. How often should turning occur? 1) every 30 minutes 2) every 2 hours 3) every 4 hours 4) every 6 hours

2) every 2 hours

July 9, 2020 A physical therapist treats a patient diagnosed with lateral epicondylitis using iontophoresis. The therapist uses dexamethasone with a current intensity of 3 mA for 20 minutes. How often during the treatment session should the therapist check the skin? 1) every minute 2) every 3-5 minutes 3) every 20 minutes 4) at the conclusion of the treatment

2) every 3-5 minutes

December 3, 2020 A physical therapist in an outpatient cardiac rehabilitation program is instructing a patient in proper lifting technique. What should the therapist recommend to help the patient avoid the Valsalva maneuver during weight lifting? 1) inhale during lifting and exhale during lowering 2) exhale during lifting and inhale during lowering 3) inhale during both 4) exhale during both

2) exhale during lifting and inhale during lowering

Nov 19, 2019 A physical therapist assesses a patient with a traumatic brain injury using the Glascow Coma Scale. If the therapist documents the eye opening score as a three (E3), which of the following responses was observed during this portion of the assessment? 1) spontaneous eye opening 2) eyes open in response to speech 3) eyes open in response to pain 4) eyes do not open (nil response)

2) eyes open in response to speech

March 20, 2021 A physical therapist prepares to use an intermittent compression device to treat a patient with lower extremity edema. The therapist would like to carefully monitor the patient during the session due to a history of mixed arterial and venous disease. Which of the following assessment procedures would be least beneficial to ensure that the patient is safely tolerating compression treatment? 1) observation of lower leg skin color 2) figure 8 3) capillary refill at great toe 4) palpation of the dorsalis pedis artery

2) figure 8

March 20, 2020 A physical therapist prepares to use an intermittent compression device to treat a patient with lower extremity edema. The therapist would like to carefully monitor the patient during the session due to a history of mixed arterial and venous disease. Which of the following assessment procedures would be least beneficial to ensure that the patient is safely tolerating the compression treatment? 1) observation of lower leg skin color 2) figure 8 measurement at the ankle 3) capillary refill time of the great toe 4) palpation of the dorsalis pedis artery

2) figure 8 measurement at the ankle

October 30, 2020 A physical therapist completes a lower quarter screening exam on a patient diagnosed with hip pathology. Assuming a normal end-feel, which of the following classifications would be most consistent with hip extension? 1) soft 2) firm 3) hard 4) empty

2) firm

May 12, 2020 A physical therapist attempts to influence the amount of scar tissue a patient develops following partial-thickness burns to their entire hand and wrist. Which of the following interventions would be the most appropriate based on the treatment goal? 1) recommend the patient wear a compression glove for 8-10 hours per day 2) fit the patient with a custom-made garment to be removed only when bathing 3) teach the patient to perform scar tissue massage 3-4 times daily 4) wrap the fingers and wrist with long-stretch bandaging daily in a spiral pattern

2) fit the patient with a custom-made garment to be removed only when bathing

March 1, 2020 A physical therapist completes a goniometric assessment of a patient's wrist. Assuming normal range of motion, which of the following motions would have the greatest available range? 1) extension 2) flexion 3) radial deviation 4) ulnar deviation

2) flexion

February 8, 2021 A patient is referred to physical therapy with a diagnosis of a grade II syndesmotic ankle sprain. What is the most common mechanism of injury for this type of ankle sprain? 1) excessive plantar flexion with inversion 2) forceful external rotation of the foot 3) excessive plantar flexion with eversion 4) vertical compression of the tibia into the talus

2) forceful external rotation of the foot

December 19, 2020 A patient explains to a physical therapist that they are scheduled to have a feeding device inserted into their stomach through an incision in the abdomen. What type of device is most likely being used in this scenario? 1) swan-ganz catheter 2) gastric tube 3) nasogastric tube 4) hickman line

2) gastric tube

Oct 31, 2019 A physical therapist reviews clinical features and diagnostic criteria associated with various types of osteogenesis imperfecta. Which of the following is included in the criteria used to delineate specific types of this condition? 1) fracture frequency 2) genetic inheritance 3) age of diagnosis 4) anticipated life expectancy

2) genetic inheritance

February 6, 2021 A patient is instructed to perform hip abd in sidelying. What muscles are targeted when performing this exercise? 1) glut max and adductor magnus 2) glut med and sartorius 3) glut med and pectineus 4) TFL and semimembranosus

2) glut med and sartorius

March 1, 2021 The physical therapy department sponsors a community education program on DM. Which of the following characteristics is not typical of type 1 diabetes? 1) age of onset less than 25 years old 2) gradual onset 3) controlled through insulin and diet 4) islet cell antibodies present at onset

2) gradual onset

Jan 30, 2020 A physical therapist reviews the medical record of a 22-year-old athlete rehabilitating from an ACL reconstruction. Which piece of objective measurement would serve as a barrier to the patient returning to the high-demand of competitive athletics? 1) 0-135 knee ROM 2) hamstrings/quad strength ratio of 50% 3) quad strength 90% of the contralateral limb 4) negative lateral pivot shift

2) hamstrings/quad strength ratio of 50%

April 15, 2020 A patient who had a lateral ankle reconstruction eight weeks ago arrives to a therapy session without their walking boot. They had previously been full weight bearing in a walking boot one week ago. The patient reports 6/10 pain over the past week with any ambulation. What would be the most appropriate action by the physical therapist? 1) have the patient return to using the walking boot as well as axillary crutches 2) have the patient use an ankle brace 3) advise the patient that an increase in pain is expected during the transition 4) refer the patient out for diagnostic imaging

2) have the patient use an ankle brace

August 9, 2020 A physical therapist treats a 6 month old infant that has a rough, slightly protruding reddish purple lesion on the right lower side of their face. What is the most appropriate term for this type of marking? 1) dermatitis 2) hemangioma 3) nevus 4) rosacea

2) hemangioma

November 7, 2020 A patient rehabilitating from a LE injury is referred to physical therapy for hydrotherapy treatments. The physical therapist would like the patient to fully extend the involved LE while sitting in the hydrotherapy tank. Which type of whirlpool would not allow the patient to extend the involved LE? 1) hubbard tank 2) highboy tank 3) lowboy tank 4) walk tank

2) highboy tank

December 26, 2020 A physical therapist observing a patient's gait identifies toeing-out during stance. This finding would most likely be associated with tightness if which of the following muscle groups? 1) ankle DF 2) hip lateral rotators 3) hip medial rotators 4) knee extensors

2) hip lateral rotators

October 16, 2020 A physical therapist attempts to transfer a dependent patient from a wheelchair to a bed. The therapist is concerned about the size of the patient, but is unable to secure another staff member to assist with the transfer. Which type of transfer would allow the therapist to move the patient with the greatest ease? 1) dependent standing pivot 2) hydraulic lift 3) sliding board 4) assisted standing pivot

2) hydraulic lift

December 16, 2020 A patient's pulmonary function tests reveal that the patient has a forced vital capacity of 4.0 liters and forced expiratory volume in 1 second (FEV1) of 3.5 liters. These results would be most anticipated with which of the following diagnoses? 1) cystic fibrosis 2) idiopathic pulmonary fibrosis 3) chronic bronchitis 4) emphysema

2) idiopathic pulmonary fibrosis

March 23, 2020 A physical therapist employed in a home health setting treats a patient following TKA. Which of the following physical therapy goals would be the most essential to avoid long-term complications? 1) decrease inflammation 2) improve ROM 3) prevent DVT 4) improve strength

2) improve ROM

September 2, 2020 A physical therapist designs an aerobic exercise program for a nine-year-old-child. Which statement best describes the heart rate and stroke volume of the child compared to an adult? 1) increased HR and SV 2) increase HR and decreased SV 3) decreased HR and increase SV 4) decreased HR and SV

2) increase HR and decreased SV

September 30, 2020 A physical therapist performs strength testing on members of a high school football team as part of a preseason assessment. The majority of the players are between the ages of 14 and 16 years of age. Which age-related phenomenon would not be characteristic of this period of development? 1) increase in muscle mass 2) increase in the number of muscle fibers 3) increase in muscle fiber size 4) increase in muscle strength

2) increase in the number of muscle fibers

March 18, 2021 The parents of a ten year old patient diagnosed with Down syndrome have been advised to discourage the child's recent interest in attempting forward rolls. Why? 1) increase risk for hydrocephalus 2) increase risk for SCI 3) increase risk for TBI 4) increase risk for developmental delay

2) increase risk for SCI

July 31, 2020 A physical therapist treats a patient rehabilitating from a lower extremity injury with a cryotherapeutic agent. Which of the finding is most accurate when using this type of intervention? 1) increased spasticity 2) increased pain threshold 3) increased nerve conduction velocity 4) increased metabolic rate

2) increased pain threshold

October 10, 2020 A patient with a transtibial prothesis using a shuttle lock suspension complains of pain in the residual limb during gait. Upon inspection, it is noted that the patient has a smaller blister at the distal end of the residual limb. What factor is most likely contributing to this finding? 1) shuttle lock not engaging the pin 2) inferior translation of the liner on the residual limb 3) prosthetic socket is fitting too tightly on the limb 4) ankle component is set into excessive dorsiflexion

2) inferior translation of the liner on the residual limb

July 14, 2020 A patient returns to physical therapy after completing medical testing using Doppler ultrasonography. What is the patient most likely presenting with? 1) peripheral neuropathy 2) intermittent claudication 3) cardiac arrhythmia 4) bursitis

2) intermittent claudication

Dec 20, 2019 A patient with complete T6 spinal cord injury works on pursed-lip breathing with a physical therapist. Based on the patient's diagnosis, which muscle(s) would the patient most likely utilize for forced expiration? 1) external intercostals 2) internal intercostals 3) diaphragm 4) quadratus lumborum

2) internal intercostals

Jan 25, 2020 A patient using a patella tendon bearing prosthesis applies a donut-shaped pad to an area of skin breakdown on the residual limb. The area of tissue damage was identified shortly after the patient-initiated prosthetic training. The therapist explains to the patient that the use of the donut-shaped pad should be avoided since it increases the: 1) ply of the prosthetic socks required for ambulation 2) ischemic effect of weight-bearing on the residual limb 3) weight-bearing forces applied to the patella tendon 4) limb volume of the residual limb

2) ischemic effect of weight-bearing on the residual limb

March 9, 2021 A physical therapist participating in a research project decides it will be necessary to utilize a relatively large sample. By including a large number of subjects, what is the researcher hoping to increase? 1) effect size 2) likelihood of rejecting a false null hypothesis 3) validity of the outcome measurements 4) reliability of the outcome measurements

2) likelihood of rejecting a false null hypothesis

October 29, 2020 A patient with systemic lupus erythematosus informs her physical therapist that she will need to cancel her scheduled therapy due to a trip. The patient reports that she is looking forward to swimming in the ocean and enjoying the beach since she believes it will help with some of her lingering joint pain. Which of the following recommendations would be the most beneficial for the patient? 1) increase the frequency of exercise due to the missed therapy visits 2) limit exposure to the sun and use sunblock for all outdoor activities 3) use cuff water weights while exercising in the water 4) avoid completing a HEP to avoid overexertion while away

2) limit exposure to the sun and use sunblock for all outdoor activities

Dec 21, 2019 A physical therapist observes that a patient has a limitation in plantar flexion range of motion on the left during ambulation activities. This mobility restriction would be most apparent during which phase of gait? 1) terminal swing 2) loading response 3) initial contact 4) midstance

2) loading response

Feb 27, 2020 A physical therapist evaluates a patient with a recent onset of low back pain consistent with a disk herniation. Pain increases with standing, lumbar motion, and coughing. Positions that are non-weight bearing alleviate the patient's symptoms. Based on the patient's clinical presentation, which of the following interventions would the therapist most likely utilize initially to address the patient's positional preferences? 1) lumbar stabilization exercises 2) lumbar traction techniques 3) lumbar flexion exercises in supine 4) lumbar extension exercises in prone

2) lumbar traction techniques

July 21, 2020 A physical therapist completes a developmental assessment on a five-month-old infant. If the therapist elects to examine the infant's palmar grasp reflex, which of the following stimuli is the most appropriate? 1) contact to the ball of the foot in upright standing 2) maintained pressure to the palm of the hand 3) noxious stimuli to the palm of the hand 4) sudden change in the position of the head

2) maintained pressure to the palm of the hand

March 5, 2021 A physical therapist is scheduled to treat a patient requiring droplet precautions? 1) gloves 2) mask 3) gloves and mask 4) gloves, gown, and mask

2) mask

April 10, 2020 A physical therapist instructs a patient to perform scar massage. Which type of lotion and frequency would be the most appropriate given the scar in the image? (Deep scar not fully healed, looks a little separated, but too old to bleed, long scar) 1) massage with vitamin E infused lotion for 5 minutes four times a day 2) massage with vitamin E infused lotion for 10 minutes two times a day 3) massage with vitamin A infused lotion for 5 minutes four times a day 4) massage with vitamin A infused lotion for 10 minutes two times a day

2) massage with vitamin E infused lotion for 10 minutes two times a day

April 22, 2020 A physical therapist assesses the pulse rate of a patient exercising on a treadmill. The therapist notes that the rhythm of the pulse is often irregular. What is the most appropriate action to ensure an accurate measurement of pulse rate? 1) select a different pulse rate 2) measure the pulse rate for 60 seconds 3) use a different stethoscope 4) document the irregular pulse rate in the patient's medical record

2) measure the pulse rate for 60 seconds

Jan 17, 2020 A patient is directed that they are only allowed to be 20% weight bearing following right knee surgery. The patient uses a scale to determine how much weight they are actually putting through the affected leg. Due to significant upper extremity weakness, the patient is only able to achieve 25% weight bearing on the affected extremity. During which phase of gait would the patient most likely be breaking their weight bearing precautions? 1) loading response 2) midstance 3) pre-swing 4) midswing

2) midstance

January 19, 2021 A physical therapist performs resisted isometric testing on a patient. The patient reports feeling pain during the test, however, strength is normal. Which of the following conclusions regarding this test is most likely? 1) severe lesion, like a fracture 2) minor lesion of a muscle or tendon 3) complete rupture of a muscle or tendon 4) intermittent claudication may be present

2) minor lesion of a muscle or tendon

March 30, 2021 A patient recovering from pnuemonia is prescribed a pharmacological agent administered with a nebulizer. Which pharmacological agent was most likely utilized? 1) Ca Channel blocker agent 2) mucolytic agent 3) nitrate agent 4) sedative hypnotic agent

2) mucolytic agent

January 2, 2020 A physical therapist treats a patient using a vacuum-assisted closure device. Which of the following best describes the mechanism used by this type of device to promote wound healing? 1) positive pressure in the wound bed 2) negative pressure in the wound bed 3) atmospheric pressure in the wound bed 4) alternating pressure in the wound bed

2) negative pressure in the wound bed

August 29, 2020 A physical therapist is reviewing the medial record of a patient with metabolic syndrome. Which risk factor is most prevalent with this diagnosis? 1) genetic origin 2) obesity/sedentary lifestyle 3) decreased thyroid hormone secretion 4) decreased blood fasting glucose level

2) obesity/sedentary lifestyle

Oct 29, 2019 A physical therapist is reviewing the medical record of a patient with metabolic syndrome. Which risk factor is most prevalent with this diagnosis? 1) genetic origin 2) obesity/sedentary lifestyle 3) decreased thyroid hormone secretion 4) decreased fasting blood glucose level

2) obesity/sedentary lifestyle

November 30, 2020 A patient seen in physical therapy for low back pain reports having a vertebroplasty performed ten weeks ago. Which of the following conditions would most likely be a part of this patient's medical history? 1) facet joint arthritis 2) osteoporosis 3) spondylotisthesis 4) spinal cord compression

2) osteoporosis

August 25, 2020 A physical therapist observes that the back height of a wheelchair extends above the patient's inferior angle of the scapula. Which finding would provide the most appropriate justification for the described wheelchair fit? 1) patient exhibits poor sitting tolerance due to pelvic obliquity 2) patient has fair trunk control, but is able to independently propel a wheelchair 3) patient requires mild to moderate lateral support due to listing or scoliosis 4) patient is unable to perform weight shifting activities in sitting due to extensor tone

2) patient has fair trunk control, but is able to independently propel a wheelchair

Jan 19, 2020 A physical therapist is employed in an acute care facility. Which scenario would provide the most appropriate justification for using restraints on a patient? 1) patient is on a medication that causes hallucinations 2) patient is too agitated to allow for the completion of an essential surgery 3) patient is verbally abusive to staff 4) patient is placed on bedrest orders following a major surgery

2) patient is too agitated to allow for the completion of an essential surgery

November 4, 2020 During an exercise session a physical therapist monitors exercise intensity with a patient post cardiac transplantation. Which method is the most appropriate for the therapist to use? 1) metabolic equivalents 2) perceived exertion scale 3) pulmonary function test 4) target HR range

2) perceived exertion scale

August 20, 2020 A physical therapist works with a patient with hemiparesis who uses a hemiplegic chair for mobility. Which activity would become more challenging for the patient based on this specific type of wheelchair? 1) reaching for objects outside the base of support 2) performing a standing transfer 3) performing independent pressure relief 4) propelling the wheelchair

2) performing a standing transfer

March 3, 2021 A physical therapist working in an acute care hospital treats a patient with transmission-based precautions. The therapist treats the patient in their room and is able to leave the door open during treatment. The therapist is required to wear a mask only once they are within 3 feet of the patient. This scenario would be most appropriate when treating a patient diagnosed with what condition? 1) TB 2) pertussis 3) impetigo 4) measles

2) pertussis

Feb 17, 2020 A physical therapist prepares to assist a patient with a sliding board transfer from a wheelchair to a bed. Which of the following directions would be most appropriate when directing the patient to perform a sliding board transfer? 1) adjust the bed height so it is 4 inches lower than the wheelchair seat 2) position the wheelchair so that it is at a slight angle to the bed 3) remove both armrests from the wheelchair before positioning the slide board 4) leave the elbows slightly bent as you move the body laterally on the slide board

2) position the wheelchair so that it is at a slight angle to the bed

Feb 8, 2020 A physical therapist performs a circulatory screening on the LE. Which location would be most appropriate for the therapist to palpate if they want to assess the posterior tibial artery pulse? 1) anterior aspect of medial malleolus 2) posterior aspect of medial malleolus 3) anterior aspect of lateral malleolus 4) posterior aspect of lateral malleolus

2) posterior aspect of medial malleolus

November 16, 2020 A physical therapist examines the output from a single lead electrocardiogram of a patient with first-degree AV heart block. What is the defining feature of this condition? 1) inverted T wave 2) prolonged PR interval 3) bizarre QRS complex 4) ST segment depression

2) prolonged PR interval

October 18, 2020 A patient reports using a wheelchair for mobility for the past 3 months following a transfemoral amputation. Assuming the goal is to have the patient become a community ambulator, which intervention would be the most beneficial to improve terminal stance range of motion during gait? 1) supine active hip flexion 2) prone lying passive hip extension 3) sidelying active hip abduction 4) supine passive hip adduction

2) prone lying passive hip extension

September 18, 2020 A physical therapist is involved in educating the family of a newborn with a brachial plexus injury sustained during birth. What is the most appropriate activity for the family? 1) perform full PROM to the shoulder in all directions 2) protect shoulder during handling and caregiving 3) perform AROM to the shoulder in all directions 4) move the UE to the waiter's tip position to avoid over stretching

2) protect shoulder during handling and caregiving

June 4, 2020 With the patient positioned in supine, a physical therapist performs an upper limb neural tension test with the arm positioned as shown in IR and wrist flexion. Based on the UE positioning what nerve is likely being tested? 1) median 2) radial 3) ulnar 4) axillary

2) radial

December 12, 2020 Which of the following is considered a benefit of administering a medication using iontophoresis instead of through oral administration? 1) enhanced systemic effects 2) reduced GI impact 3) diminished skin sensitivity 4) increased cost effectiveness

2) reduced GI impact

September 25, 2020 A physical therapist prepares to instruct a patient in pursed-lip breathing. Which of the following would be the most desired outcome of this intervention? 1) increased chest wall mobility 2) relief of dyspnea 3) symmetrical chest wall expansion 4) improved cough effectiveness

2) relief of dyspnea

October 7, 2020 A physical therapist assess both power and strength in a 75-year-old patient with sarcopenia. Which of the following exercises would best address the loss of power the therapist observed during the patient's performance of functional activities? 1) progressive walking program with light weights 2) resistance exercises with a rapid concentric phase 3) progression of static to dynamic balance exercises 4) wall squats with isometric hold at 90 degrees of hip flexion

2) resistance exercises with a rapid concentric phase

Dec 12, 2019 A patient has limited finger flexion and difficulty with gripping objects. The therapist notes limitation in the patient's range of motion for DIP joint flexion when the PIP joint is held in extension. However, the patient has normal DIP joint flexion when the PIP joint is partially flexed. What structure is most likely limiting the patient's range of motion? 1) DIP joint capsule 2) retinacular ligaments 3) extensor digitorum muscle 4) FDP muscle

2) retinacular ligaments

March 29, 2021 An entry in the medical record indicates that a patient is experiencing tricuspid valve regurgitation. Which location would have increased blood volume based on the valvular pathology? 1) left atrium 2) right atrium 3) left ventricle 4) right ventricle

2) right atrium

September 19, 2020 An entry in the medical record indicates that a patient is experiencing tricuspid valve regurgitation. Where would this condition most likely result in an increased volume of blood? 1) left atrium 2) right atrium 3) left ventricle 4) right ventricle

2) right atrium

July 28, 2020 A phyiscal therapist assesses a 1-month-old infant. During the treatment session, the therapist strokes the cheek of the infant causing the infant to turn its mouth towards the stimulus. What is this action utilized to asses? 1) moro reflex 2) rooting reflex 3) startle reflex 4) righting reflex

2) rooting reflex

March 11, 2021 A patient is referred to physical therapy following a Bankart repair. Which of the following interventions should the physical therapist most anticipate needing to incorporate into the early phase of rehabilitation based on this diagnosis? 1) ROM emphasizing shoulder lateral rotation 2) scapular and glenohumeral dynamic stabilization exercises 3) chest press and lat pull down 4) grade IV joint mobs emphasizing anterior glide

2) scapular and glenohumeral dynamic stabilization exercises

Nov 12, 2019 A patient is referred to physical therapy following a Bankart repair. Which of the following interventions should the physical therapist most anticipate needing to incorporate into the patient's plan of care based on this diagnosis? 1) range of motion exercises emphasizing shoulder lateral rotation 2) scapular and glenohumeral dynamic stabilization exercises 3) chest press and latissimus pull-down exercises 4) grade IV joint mobilizations emphasizing anterior glides

2) scapular and glenohumeral dynamic stabilization exercises

Dec 11, 2019 A physical therapist preparing a hot pack notices the water in the hot pack unit is cloudy. The most probable explanation is: 1) power failure 2) seepage from a hot pack 3) ineffective heating element 4) thermostat set too low

2) seepage from a hot pack

April 13, 2020 An 85-year-old female patient was recently diagnosed with osteoporosis after a fall in which she sustained a femoral neck fracture. The patient reports that her lifestyle has become much more sedentary over the past five years. Which type of osteoporosis is most consistent with the presented scenario? 1) idiopathic 2) senile 3) postmenopausal 4) secondary

2) senile

November 14, 2020 A physical therapist examines a patient diagnosed with post-polio syndrome. Which of the following areas is the least likely to be affected based on the patient's diagnosis? 1) strength 2) sensation 3) endurance 4) functional mobility

2) sensation

OCt 27, 2019 A physical therapist examines a patient diagnosed with post-polio syndrome. Which of the following areas is the least likely to be affected based on the patient's diagnosis? 1) strength 2) sensation 3) endurance 4) functional activity

2) sensation

May 18, 2020 A patient presents with a mixture of superficial partial-thickness and deep partial-thickness burns along the anterior surface of the right upper extremity. A culture of the exudate from the wound is positive for Pseudomonas infection. Which of the following topical medications would most likely be utilized in conjunction with wound dressings? 1) bacitracin 2) silver sulfadiazine 3) collagenase 4) nitrofurazone

2) silver sulfadiazine

November 26, 2020 A physical therapist instructs a 55-year-old patient with significant bilateral lower extremity paresis to transfer from a wheelchair to a mat table. The patient has normal upper extremity strength and has no other known medical problems. What is the most appropriate transfer technique? 1) dependent squat pivot transfer 2) sliding board transfer 3) two-person lift 4) hydraulic lift

2) sliding board transfer

November 21, 2020 Which of the following may assist a child with cerebral palsy during a stand pivot transfer? 1) primitive reflexes 2) spasticity 3) athetosis 4) equinovarus contracture

2) spasticity

November 6, 2020 A physical therapist discusses positioning issues with the nursing staff prior to a scheduled treatment session. What bony prominance would be particularly susceptible to breakdown with the patient positioned in the supine position? 1) anterior shldr 2) spinous processes 3) patella 4) anterior superior iliac spines

2) spinous processes

June 15, 2020 A physical therapist examines a patient with a history of respiratory complaints. The therapist is unable to determine from the history taking if the patient has an obstructive or restrictive lung disease. Which of the following assessments would be the most helpful in categorizing the patient's lung dysfunction? 1) bronchoscopy 2) spirometry 3) respiration rate 4) auscultation of lung sounds

2) spirometry

September 13, 2020 A physical therapist observes a burn on the dorsal surface of a patient's arm. The wound area is mottled red with a number of blisters. The therapist informs the patient that healing should take place in less than 3 weeks. What is the description most indicative of? 1) superficial burn 2) superficial partial-thickness burn 3) deep partial-thickness burn 4) full-thickness burn

2) superficial partial-thickness burn

June 12, 2020 A physical therapist prescribes a wheelchair for a patient with bilateral lower amputations. Which wheelchair feature would be most essential for this patient? 1) friction surface handrails 2) the drive wheels are set behind the vertical back supports 3) reclining back with elevating leg rests 4) removable arm rests

2) the drive wheels are set behind the vertical back supports

July 8, 2020 A physical therapist measures elbow flexion while a patient grasps the handgrip of a walker in standing. The therapist records elbow flexion as 35 degrees. Which statement best describes the height of the walker? 1) the walker height is too low 2) the walker height is too high 3) the walker height is appropriate 4) not enough information given

2) the walker height is too high

Oct 30, 2019 A physical therapist measures elbow flexion while a patient grasps the handgrip of a walker in standing. The therapist records elbow flexion as 35 degrees. Which statement best describes the height of the walker? 1) the walker height is too low for the patient 2) the walker height is too high for the patient 3) the walker height is appropriate for the patient 4) not enough information is given to assess walker height

2) the walker height is too high for the patient

Jan 20, 2020 A physical therapist examines the posture of a patient with ankylosing spondylitis using a plumb line. From the lateral view, which of the following would be considered an abnormal position for the plumb line give the likely postural deformity associated with this pathology? 1) through the external auditory meatus 2) through the bodies of the thoracic vertebrae 3) slightly posterior to the hip joint 4) slightly anterior to the knee joint

2) through the bodies of the thoracic vertebrae

Jan 12, 2020 A physical therapist makes wheelchair recommendations for a patient with a C4 complete spinal cord injury. Which wheelchair feature would be the most desirable for this patient? 1) removable armrests 2) tilt-in-space frame 3) handrims with rim projections 4) handheld joystick controls

2) tilt-in-space frame

July 29, 2020 A physical therapist treats a patient post MI that has completed an inpatient cardiac program and has been cleared for exercise in an outpatient cardiac rehab program. Which training intensity would be most likely for this patient? 1) training intensity at 10%-30% VO2 max with a RPE between 11-13 2) training intensity at 40%-60% VO2 max with a RPE between 11-13 3) training intensity at 10%-30% VO2 max with a RPE between 14-16 4) training intensity at 40%-60% VO2 max with a RPE between 14-16

2) training intensity at 40%-60% VO2 max with a RPE between 11-13

May 5, 2020 A physical therapist presents an inservice on handling techniques for patients with osteogenesis imperfecta. When handling the patient, which type of osteogenesis imperfecta would require the most caution by the therapist? 1) type 1 2) type 2 3) type 3 4) type 4

2) type 2

April 14, 2020 A physical therapist assesses a patient's pulse by palpating between the flexor digitorum superficialis and the flexor carpi ulnaris tendons at the wrist. This site is used to assess the: 1) radial artery 2) ulnar artery 3) brachial artery 4) carotid artery

2) ulnar artery

December 15, 2020 A physical therapist is guarding a patient that is descending the stairs and is positioned in front of the patient. The patient is using a gait belt but has a loss of balance in a forward direction. What would be the most immediate therapist response? 1) pull back on the gait belt and pull back on the trunk 2) use 1 hand to press back against the chest, and instruct to look up and straighten the trunk 3) use 1 hand to press fwd against the post pelvis or posterior trunk while securing the gait belt 4) pull fwd on the gait belt and slowly lower the patient to sit on the step if needed

2) use 1 hand to press back against the chest, and instruct to look up and straighten the trunk

March 30, 2020 A physical therapist plans to use a TENS device for a patient with chronic pain. Which of the following is an absolute contraindication? 1) Use on the periphery of an open wound 2) use over the muscles of the anterolateral neck 3) use on a patient with premature atrial contractions 4) use in the presence of arteriosclerosis

2) use over the muscles of the anterolateral neck

January 14, 2021 A physical therapist instructs a patient post fractured left hip in gait training activities. The patient is weight bearing as tolerated and uses a large base quad cane. Which of the following techniques would be the most appropriate? 1) using cane on left with the longer legs facing away 2) using cane on right with longer legs facing away 3) using cane on left with longer legs facing in 4) using cane on right with longer legs facing in

2) using cane on right with longer legs facing away

February 2, 2021 A patient with a transfemoral amputation is assigned a K-level of 1. Based on the K-level assignment, which functional task would represent the patient's highest anticipated level of function? 1) propelling a manual wheelchair 2) walking short distances in the home 3) negotiation of 6 stairs 4) walking down a sloped driveway to the mailbox

2) walking short distances in the home

January 11, 2021 A patient who has vascular intermittent claudication is seen for cardiac rehab. Which ABI value would be the most consistent with intermittent claudication symptoms that only occur during fairly intense exercise? 1) .35 2) .55 3) .75 4) 1.1

3) .75

Feb 6, 2020 A group of physical therapists employed in an acute care hospital is responsible for developing departmental guidelines for electrical equipment care and safety. What is the minimum required testing interval for electrical equipment? 1) 3 months 2) 6 months 3) 12 months 4) 24 months

3) 12 months

February 16, 2021 Before beginning to train for a wheelchair racing event, a 40 y.o pt with complete C7 tetraplegia undergoes a graded exercise test using an UBE. In the absence of cardiac pathology, which value represents the most likely maximum heart rate? 1) 60 bpm 2) 80 bpm 3) 120 bpm 4) 180 bpm

3) 120 bpm

October 27, 2020 A physical therapist initiates a treadmill exercise program designed to promote cardiovascular endurance. The patient's systolic blood pressure was recorded at rest 125 mmHG. What systolic blood pressure value would be most anticipated when the patient exercises at 3.0 metabolic equivalents? 1) 115 mmHg 2) 130 mmHg 3) 152 mmHg 4) 170 mmHg

3) 152 mmHg

February 25, 2021 A physical therapist assesses a patient's muscle tone following a stroke and reports that while the patient has marked tone throughout the range of motion, their extremity can still be easily moved. Using the Modified Ashworth Scale, which grade of spasticity would be most appropriate? 1) 1 2) 1+ 3) 2 4) 3

3) 2

May 26, 2020 A physical therapist assesses a patient's muscle tone following a stroke and reports that while the patient has marked tone throughout the ROM, their extremity can still be easily moved. Using the modified ashworth scale, which grade of spasticity would be the most appropriate for this patient? 1) 1 2) 1+ 3) 2 4) 3

3) 2

March 15, 2020 A patient who recently had a TKA plan to ask the surgeon about their ability to drive at an upcoming visit. Which patient would be the best candidate to begin driving? 1) 1 week post R TKA currently taking Tylenol 2) 2 weeks post-op taking tramadol 3) 2 weeks post-op taking meloxicam 4) 3 weeks post-op taking meperidine

3) 2 weeks post-op taking meloxicam

October 17, 2020 A physical therapist contemplates the necessary amount of force to overcome friction when performing mechanical lumber traction using a non-split traction table. What percentage of a patient's body weight would be most representative of this value? 1) 5% 2) 15% 3) 25% 4) 50%

3) 25%

December 31, 2020 A patient with diabetes is being treated for a wound on the plantar surface of the foot. The wound is documented as a deep ulcer with abscess. Which of the following numeric grades would the physical therapist use when grading this ulcer based on the Wagner Grade Classification System? 1) 1 2) 2 3) 3 4) 5

3) 3

Feb 9, 2020 A patient with diabetes is being treated for a wound on the plantar surface of the foot. The wound is documented as a deep ulcer with abscess. Which of the following numeric grades would the physical therapist use when grading this ulcer based on the Wagner Grade Classification system? 1) 1 2) 2 3) 3 4) 4

3) 3

March 16, 2021 A physical therapist decides to perform a caudal glide hip distraction mobilization technique on a patient experiencing hip pain. Which of the following hip positions is the most appropriate to initially perform this technique? 1) 20 ext, 15 abd, full IR 2) 20 flex, 15 abd, slight IR 3) 30 flex, 30 abd, slight ER 4) neutral flex, neutral abd, slight ER

3) 30 flex, 30 abd, slight ER

October 6, 2020 A physical therapist treats an 80-year-old patient with general deconditioning following a recent hospitalization for pneumonia. Which test would provide the therapist with the best assessment of the patient's muscular endurance? 1) TUG 2) gait speed over 10 ft 3) 30 second sit to stand 4) sharpened romberg

3) 30 second sit to stand

Jan 18, 2020 A physical therapist works with a nine-year-old child with cystic fibrosis. The age which best approximates the median age of survival for a patient with cystic fibrosis is: 1) 15 2) 25 3) 35 4) 50

3) 35

December 22, 2020 A physical therapist participates in a community health fair by screening elderly patients to determine their risk for falling. The therapist utilizes the Berg Balance Scale for the screening and therefore is required to rate patients on 14 specific items. When using the Berg, what type of scale should be used to score each of these items? 1) 3-point scale 2) 4-point scale 3) 5-point scale 4) 6-point scale

3) 5-point scale

June 7, 2020 A physical therapist uses a functional electrical stimulation as part of a treatment regimen designed to improve quad strength. Which on:off time ratio would result in the most rapid onset of muscle fatigue? 1) 3:1 2) 1:4 3) 5:1 4) 1:6

3) 5:1

Oct 25, 2019 A physical therapist uses functional electrical stimulation as part of a treatment regimen designed to improve quadriceps strength. Which on:off time ratio would result in the most rapid onset of muscle fatigue? 1) 3:1 2) 1:4 3) 5:1 4) 1:6

3) 5:1

May 25, 2020 A physical therapist determines a patient's heart rate by counting the number of QRS complexes in a 6 second EKG strip. Assuming the therapist identifies 8 QRS complexes in the strip, what should the HR be recorded as? 1) 40 bpm 2) 60 bpm 3) 80 bpm 4) 100 bpm

3) 80 bpm

January 25, 2021 A physician instructs a patient to utilize a knee derotation brace for all athletic activities. Which of the following conditions would be most warrant the use of this type of brace? 1) medial meniscus repair 2) ACL reconstruction 3) ACL insufficiency 4) PCL reconstruction

3) ACL insufficiency

Dec 23, 2019 A physical therapist interviews a patient recently involved in a motor vehicle accident, in which the patient sustained multiple lower extremity injuries. During the interview, the patient appears to be very depressed and withdrawn. Which of the following would be least likely to encourage active dialogue by the patient? 1) How does your knee feel today? 2) What are your goals for therapy? 3) Do you have trouble sleeping at night? 4) Tell me about your present condition?

3) Do you have trouble sleeping at night?

Dec 5, 2019 A physical therapist performs a muscle length assessment on a patient's gastroc and determines that shortening of this muscle only permits five degrees of ankle dorsiflexion. Which of the following gait deviations should the therapist most anticipate based on this finding? 1) Foot slap at heel strike (initial contact) 2) Toe down instead of heel strike 3) Heel lift during stance 4) No toe off during toe off (pre-swing)

3) Heel lift during stance

Nov 10, 2019 A physical therapist applies a hot pack to a patient's hamstring region prior to performing soft tissue mobilization and stretching. Assuming the goal was to heat the tissue prior to the interventions, which of the following medications would most likely limit this desired effect? 1) Amlodipine (Norvasc) 2) Cyclobenzaprine (Flexeril) 3) Pseudoephedrine (Sudafed) 4) Nitroglycerin (Nitrostat)

3) Pseudoephedrine (Sudafed)

July 15, 2020 A patient reports muscle pains of excess severity in relation to their current activity level and the interventions provided. Which of the following medications would most likely be associated with this finding? 1) amiodipine (Norvasc) 2) albuterol (Ventolin) 3) Simvastatin (Zocor) 4) methotrexate (Rheumatrex)

3) Simvastatin (Zocor)

December 29, 2020 A primary care physician examines a 10 y.o patient with proximal weakness globally and suspects the patient may have muscular dystrophy. The physician refers the patient to a neurologist to determine if the patient has Becker muscular dystrophy of DMD. Which of the following factors would most likely lead to a diagnosis of Becker muscular dystrophy over DMD? 1) calf muscle hypertrophy 2) increased lumbar lordosis 3) absence of Trendelenburg gait 4) presence of scoliosis

3) absence of Trendelenburg gait

Nov 5, 2019 A patient rehabilitating form a lower extremity injury is instructed to perform the wall sits. This exercise would most likely be prescribed to: 1) address balance impairments 2) address postural deviation 3) address muscular endurance impairments 4) address range of motion impairments

3) address muscular endurance impairments

July 12, 2020 A patient with asthma uses her rescue inhaler 15 minutes prior to a scheduled therapy session. Which of the following medications was most likely used to prevent exercise-induced dyspnea? 1) fexofendadine (allegra) 2) acetycyeine (mucomyst) 3) albuterol (ventolin) 4) tiotropium (spiriva)

3) albuterol (ventolin)

Nov 18, 2019 A physical therapist prepares to quantify the amount of elbow flexion for a patient rehabilitating from a radial head fracture. Which of the following steps would be the last to occur when measuring range of motion with a goniometer? 1) stabilize the proximal joint segment 2) determine end-feel 3) align the goniometer 4) palpate bony landmarks

3) align the goniometer

Nov 22, 2019 A physical therapist assesses a patient with suspected hip pathology. Which individual would inherently have the greatest hip stability? 1) a child with 20 degrees of femoral anteversion 2) a child with 30 degrees of femoral anteversion 3) an adult with 5 degrees of femoral anteversion 4) an adult with 25 degrees of femoral anteversion

3) an adult with 5 degrees of femoral anteversion

Jan 1, 2020 A physical therapist treats a patient status post myocardial infarction. The physician prescribes moderate exercise activities no greater than 5 METs. Which of the following descriptions would be most accurate for an activity that requires 5 METs? 1) an increase in systolic blood pressure of 25 mmHG above resting value 2) an increase in heart rate of 50 beats per minute above the resting value 3) an increase in oxygen uptake five times greater than the resting value 4) an increase in pulse pressure of 10 mmHG above the resting value

3) an increase in oxygen uptake five times greater than the resting value

Dec 3, 2019 A physical therapist prepares to perform a joint mobilization on a patient with adhesive capsulitis. When applying the convex-concave rule, which direction should the mobilizing force be applied to address the greatest restriction of range of motion expected based on the presence of a capsular patter at the glenohumeral joint? 1) posterior glide 2) inferior glide 3) anterior glide 4) superior glide

3) anterior glide

July 17, 2020 A physical therapist observes a patient complete hip abduction and adduction exercises in standing. Which axis of motion is utilized with these particular motions? 1) frontal 2) vertical 3) anterior-posterior 4) longitudinal

3) anterior-posterior

Dec 29, 2019 A physical therapist discusses the importance of a well-balanced diet with a patient diagnosed with type 2 diabetes mellitus. The most appropriate action to emphasize the importance of diet is: 1) provide a handout from the American Diabetes Association which outlines an appropriate diet 2) ask other patients that have made dietary changes to speak to the patient 3) arrange for a consultation with a dietician 4) provide copies of recent research articles which cite the benefit of a well-balanced diet

3) arrange for a consultation with a dietician

August 24, 2020 A patient presents with a small, round, partial-thickness wound on the lateral malleolus with distinct wound edges. Upon examination, the wound bed is a grey color without evidence of granulation tissue and appears to be dry. The wound is extremely tender to palpation and patient reports calf pain during walking. These findings are most associated with which of the following types of ulcers? 1) venous 2) neuropathic 3) arterial 4) decubitus

3) arterial

November 29, 2020 A patient presents with a small, round, partial-thickness wound on the lateral malleolus with distinct wound edges. Upon exam, the wound appears to be grey and dry. The wound is extremely tender and the pt reports calf pain during walking. Which type of ulcer? 1) venous stasis 2) neuropathic 3) arterial 4) decubitus

3) arterial

June 10, 2020 A patient with Alzheimer's disease is referred to physical therapy for instruction in an exercise program. What is the most appropriate intial step? 1) provide verbal and written instructions 2) frequently repeat multiple step directions 3) assess the patient's cognitive status 4) avoid using medical terminology

3) assess the patient's cognitive status

Dec 25, 2019 While participating in a soccer game, a player complains of gradually increasing shortness of breath and eventually begins to experience wheezing and dyspnea. The player is removed from the game so that they can take medication to control their asthma attack. Which type of medication was most likely administered? 1) mucolytics 2) antihistamines 3) beta-adrenergic agonists 4) antitussives

3) beta-adrenergic agonists

June 5, 2020 While participating in a soccer game, a player complains of gradually increasing shortness of breath and eventually begins to experience wheezing and dyspnea. The player is removed from the game so that they can take medication to control their asthma attack. Which type of medication was most likely administered? 1) mucolytics 2) antihistamines 3) beta-adrenergic agonists 4) antitussives

3) beta-adrenergic agonists

October 9, 2020 A patient rehabilitating from a radial head fracture performs progressive resistive exercises designed to strengthen the forearm supinators. Which muscle would be of particular importance to achieve the desired outcome? 1) brachialis 2) brachioradialis 3) biceps brachii 4) anconeus

3) biceps brachii

August 21, 2020 A physical therapist reviews the medical record of a patient with an arthritic condition. What piece of information would be most useful to definitively diagnose rheumatoid arthritis? 1) pain profile 2) joint symptoms 3) blood tests 4) age on onset

3) blood tests

November 8, 2020 A physical therapist obtains a complete medial history prior to administering cryotherapy. Which condition would not be considered a contraindication to cryotherapy? 1) Raynaud's disease 2) cryoglobulinemia 3) cancer 4) cold urticaria

3) cancer

August 12, 2020 A physical therapist prescribes a wheelchair with large diameter handrims for a patient. Which diagnosis would be most likely to benefit from this type of wheelchair? 1) C4 SCI 2) T1 SCI 3) central cord syndrome 4) posterior cord snydrome

3) central cord syndrome

Nov 24, 2019 A physical therapist gathers data to assist in differential diagnoses. The therapist determines that the patient's present pain is located primarily in the right posterior shoulder region with intermittent paresthesias in the right middle finger. This type of clinical presentation is most consistent with which of the following conditions? 1) AC joint arthritis 2) biceps tendinitis 3) cervical radiculopathy 4) rotator cuff tear

3) cervical radiculopathy

November 1, 2020 A physical therapist gathers data to assist in differential diagnosis. The therapist determines that the patient pain is located primarily in the posterior shoulder region. What is the type of clinical presentation most consistent with? 1) AC joint arthritis 2) biceps tendinitis 3) cervical radiculopathy 4) rotator cuff tear

3) cervical radiculopathy

October 22, 2020 A physical therapist administers phonophoresis to a patient rehabilitating from medial epicondylitis. Which action best explains the effectiveness of ultrasound in drug delivery? 1) changes in systemic blood flow 2) changes in pH of the skin 3) changes in permeability of the skin 4) changes in temperature of the skin

3) changes in permeability of the skin

July 16, 2020 A physical therapist assesses a patient's present pain level and concludes that the current patient-controlled analgesia protocol is not adequate. What is the most appropriate action? 1) modify the allowable medication dosage 2) eliminate the lockout interval 3) contact the patient's nurse 4) page the patient's referring physician

3) contact the patient's nurse

Nov 4, 2019 A physical therapist assesses a patient's present pain level and concludes that the current patient-controlled analgesia protocol is not adequate. The most appropriate action is too: 1) modify the allowable medication dosage 2) eliminate the lockout interval 3) contact the patient's nurse 4) page the patient's referring physician

3) contact the patient's nurse

May 23, 2020 A physical therapist performs a cardiovascular system review on a 78-year-old female referred to physical therapy for instruction in an exercise program. Prior to initiating exercise, the physical therapist records the patient's respiratory rate as 23 bpm and blood pressure as 165/97 mmHg at rest. Based on the values what should the PT do? 1) instruct the patient in the exercise program 2) instruct patient in an HEP limited to less than 3 METs 3) contact the referring MD to discuss obtained values 4) contact emergency medical services

3) contact the referring MD to discuss obtained values

Nov 7, 2019 A physical therapist treats a patient rehabilitating from a stroke involving the right hemisphere secondary to an anteriovenous malformation. Based on the presented image (therapist helping patient pick up ball and move it laterally). What is the therapist working on with this patient? 1) mobility 2) stability 3) controlled mobility 4) skill

3) controlled mobility

Jan 11, 2020 A physical therapist reads in the medical record that a wound resulting from venous insufficiency was classified as "Black" using the red-yellow-black system. Given the wound's classification, which of the following interventions would be most appropriate? 1) protect the wound with a transparent film dressing 2) using negative pressure wound therapy on the wound 3) debride the wound with forceps 4) avoid intervening until the wound is classified as "yellow"

3) debride the wound with forceps

March 31, 2021 A physical therapist reads in the medical record that a wound resulting from venous insufficiency was classified as "black" using the red-yellow-black system. Given the wound's classification, which of the following interventions would be the most appropriate? 1) protect the wound with a transparent film dressing 2) use negative pressure wound therapy on the wound 3) debride the wound with forceps 4) avoid intervening until the wound is classified as "yellow"

3) debride the wound with forceps

March 4, 2020 A patient rides a stationary bike at 50 rpm following knee surgery. The physical therapist would like to provide more of a stretch to the patient's knee while cycling to facilitate flexion ROM. Which of the following modifications to the cycling parameters would be the most appropriate to achieve the therapist's objective? 1) decrease the seat height and increase the revolutions per minute 2) increase the seat height and increase the revolutions per minute 3) decrease the seat height and decrease the revolutions per minute 4) increase the seat height and decrease the revolutions per minute

3) decrease the seat height and decrease the revolutions per minute

December 25, 2020 A physical therapist observes the gait of a patient four weeks status post R TKA. Which gait description would be most likely? 1) increased cadance 2) increased circumduction of R limb 3) decreased single support phase on R 4) decreased double support phase

3) decreased single support phase on R

September 28, 2020 A physical therapist transports a patient in a wheelchair to the parallel bars in preparation for ambulation activities. The patient is s/p abdominal surgery and has not ambulated in over 2 weeks. What is the most appropriate action to facilitate ambulation? 1) assist the patient to standing 2) monitor the patient's vital signs 3) demonstrate the ambulation in the parallel bars 4) secure an additional staff member to offer assistance

3) demonstrate the ambulation in the parallel bars

October 26, 2020 A physical therapist observes a patient complete a test that requires the patient to perform quick alternating pronation and supination of the forearm. What can this test be used to identify? 1) dysmetria 2) dysphagia 3) dysdiadochokinesia 4) dysarthria

3) dysdiadochokinesia

December 2, 2020 A physical therapist observes a patient's breathing pattern prior to initiating an exercise session. The therapist concludes that the rate and rhythm are within normal limits. What is this type of breathing best termed as? 1) apnea 2) dyspnea 3) eupnea 4) orthopnea

3) eupnea

January 18, 2021 A physical therapist performs gait training with a patient outdoors to stimulate the uneven terrain that the patient will encounter upon discharge. The therapist monitors the patient closely due to extreme heat and humidity. What is the primary mode of loss during exercise? 1) conduction 2) convection 3) evaporation 4) radiation

3) evaporation

April 30, 2020 A patient recently diagnosed with a deep vein thrombophlebitis is placed on heparin. What is the primary side effect? 1) hypotension 2) depression 3) excessive anticoagulation 4) thrombocytopenia

3) excessive anticoagulation

September 17, 2020 A physical therapist observes the gait of a patient referred to physical therapy for git training following a transtibial amputation. The therapist concludes that the patient exhibits decreased step length bilaterally. What is the most likely rationale associated with this finding? 1) joint contracture 2) muscle weakness 3) fear and insecurity 4) fatigue

3) fear and insecurity

Nov 27, 2019 A physical therapist places a four-month-old infant onto their stomach to assess developmental milestones. If the tonic labryrinthine reflex is still present, what response would the therapist most expect to observe in the prone position? 1) flexion of the arms and extension of the legs 2) extension of the arms and flexion of the legs 3) flexion of the arms and the legs 4) extension of the arms and the legs

3) flexion of the arms and the legs

May 15, 2020 A physical therapist examines the wound of a patient who sustained a burn on their anterior thigh. The wound bed exhibits areas of charred white leathery tissue and the therapist is able to visualize damage to the subcutaneous fat layer. Which burn classification is most consistent with this clinical presentation? 1) superficial partial-thickness 2) deep partial-thickness 3) full-thickness 4) subdermal

3) full-thickness

March 28, 2021 A PT performs a cranial nerve test by assessing a patient's ability to distinguish sour and bitter tastes on the posterior third of the tongue. Assuming the patient is unable to accurately identify the tasted substances, which of the following cranial nerves is likely damaged? 1) facial 2) trigeminal 3) glossopharyngeal 4) hypoglossal

3) glossopharyngeal

March 26, 2021 A patient exhibits limited and pain-free active shoulder external rotation, however, passive ER is normal. Resistive testing of the shoulder is weak and pain free. Which of the following conditions is most consistent with findings? 1) adhesion of anterior joint capsule 2) fear-related psychological response 3) grade III strain of the infraspinatus muscle 4) grade I strain of the shoulder ER

3) grade III strain of the infraspinatus muscle

Feb 24, 2020 A patient with a history of a grade III anterior talofibular sprain presents with a capsular pattern of the talocrural joint. Based on this finding, which of the following interventions would be the most appropriate to address this residual impairment? 1) closed-chain proprioception exercises for the involved lower extremity 2) progression of strengthening exercises for the fibularis longus and brevis 3) grade IV joint mobilizations emphasizing anterior glides of talus 4) grade III joint mobilizations emphasizing posterior glides of the talus

3) grade IV joint mobilizations emphasizing anterior glides of talus

Feb 19, 2020 A patient with a history of a grade III anterior talofibular sprain presents with a capsular pattern of the talocrural joint. Based on this finding, which of the following interventions would be the most appropriate to address this residual impairment? 1) closed-chain proprioceptive exercises for the involved lower extremity 2) progression of strengthening exercises for the fibularis longus and brevis 3) grade IV joint mobilizations emphasizing anterior glides of the talus 4) grade III joint mobilizations emphasizing posterior glides of the talus

3) grade IV joint mobilizations emphasizing anterior glides of the talus

October 12, 2020 A patient in an acute care hospital has a catheter inserted into the internal jugular vein. The catheter travels through the superior vena cava and into the right atrium. The device permits removal of blood samples, administration of medication, and monitoring of central venous pressure. What is the most appropriate term for this device? 1) arterial line 2) central venous pressure catheter 3) hickman catheter 4) swan-ganz catheter

3) hickman catheter

Initially the therapist flexes the patient's knee and the patient performs and isometric contraction. The therapist then assists the leg into knee extension, and encourages the patient to return the leg to the shortened position using an isotonic contraction. Which PNF technique is the therapist using? 1) contract-relax 2) normal timing 3) hold-relax active movement 4) slow reversal

3) hold-relax active movement

March 14, 2020 A patient reports brief episodes of significant dizziness when standing during the last 24 hours. The patient states that her symptoms began in conjunction with a change in the diuretic medication she takes for her congestive heart failure. The most likely rationale for this new symptom is: 1) hypertension due to a decrease in blood plasma volume 2) hypertension due to an increase in blood plasma volume 3) hypotension due to a decrease in blood plasma volume 4) hypotension due to an increase in blood plasma volume

3) hypotension due to a decrease in blood plasma volume

Jan 28, 2020 A patient diagnosed with T5 paraplegia is discharged from a rehab hospital following 16 weeks of therapy. Assuming a normal recovery, which of the following most accurately describes that status of the patient's bathroom transfers? 1) independent with the presence of an attendant 2) independent with a sliding board 3) independent with bathroom adaptations 4) independent

3) independent with bathroom adaptations

Feb 29, 2020 A patient diagnosed with T5 paraplegia is discharged from a rehabilitation hospital following 16 weeks of therapy. Assuming a normal recovery, which of the following most accurately describes the status of the patient's bathroom transfers? 1) independent with the presence of an attendant 2) independent with sliding board 3) independent with bathroom adaptions 4) independent

3) independent with bathroom adaptions

Feb 25, 2020 A patient diagnosed with fibromyalgia syndrome reports difficulty with managing flare-ups of their symptoms. Which of the following interventions should the physical therapist most emphasize in order to address the patient's compliant? 1) lumbar stabilization exercises 2) desensitization techniques 3) instruction in pacing exercises 4) general flexibility training

3) instruction in pacing exercises

April 21, 2020 A patient presents to physical therapy following surgery for a T10 vertebral compression fracture. The physical therapist views the patinet's x-rays and notes that instrumentation was not used as part of the surgery. Which of the following surgical procedures was most likely performed? 1) spinal fusion 2) laminectomy 3) kyphoplasty 4) foraminotomy

3) kyphoplasty

December 9, 2020 A physical therapist prepares to examine a patient with a suspected grade III sprain of the anterior cruciate ligament. Which of the following special tests is considered to have the highest sensitivity for detecting this type of injury in the acute phase? 1) anterior drawer 2) mcmurray 3) lachman 4) pivot shift

3) lachman

July 30, 2020 A physical therapist works on gait training with a patient and observes an ataxic gait pattern. Which of the following medications would be the most likely to negatively influence balance during gait activities? 1) prednisolone 2) methotrexate 3) lorazepam 4) levadopa

3) lorazepam

January 24, 2021 A patient post radial head fracture has developed an elbow flexion contracture. Which of the following interventions is considered a passive exercise technique to increase range of motion? 1) contract relax 2) hold relax 3) maintained pressure 4) rhythmic stabilization

3) maintained pressure

Jan 27, 2020 A physical therapist measures a patient for a straight cane prior to beginning ambulation activities. Which gross measurement method would provide the best estimate of cane length? 1) measuring from the head of the fibula straight to the floor and multiplying by 2 2) measuring from the iliac crest straight to the floor 3) measuring from the greater trochanter straight to the floor 4) dividing the patient's height by 2 and adding 3 inches

3) measuring from the greater trochanter straight to the floor

April 7, 2021 Members of a health promotion task force design a program that will screen individuals in selected retirement communities annually for osteoporosis. Which screening tool would be the most cost effective and reliable to incorporate as part of the program? 1) physical activity survey 2) dietary analysis 3) measuring height 4) urinalysis screening

3) measuring height

November 9, 2020 A patient 8 days ACL reconstruction using a patellar tendon autograft is examined in physical therapy. Which of the following exercises would be the most appropriate based on the patient's post-op status? 1) limited range isokinetics at 30 degrees per second 2) unilateral leg press 3) mini-squats in standing 4) active knee ext in short sitting

3) mini-squats in standing

January 12, 2021 A patient scheduled to undergo a transtibial amputation of the left lower extremity. In addition, the patient is one month post right total knee arthroplasty. Given the patient's past and current surgical history, the physical therapist should expect which of the following activities to be the most difficult for the patient following their amputation? 1) rolling from supine to sidelying 2) moving from supine to sit 3) moving from sitting to standing 4) ambulating in the parallel bars

3) moving from sitting to standing

A physical therapist reviews the current pain management protocol of a patient three days post TKA. Which of the following medications would the patient most likely have been prescribed for breakthrough pain as needed? 1) Acetaminophen (1,000 mg) 2) meloxicam (15 mg) 3) oxycodone (5 mg) 4) tramadol (100 mg)

3) oxycodone (5 mg)

July 7, 2020 A physical therapist prepares to treat a patient for trigger points in their right upper trapezius muscle using a vapocoolant spray. Which of the following would be the correct procedure for positioning the patient before applying the spray? 1) have the patient actively side bend their neck to the left 2) have the patient actively side bend their neck to the right 3) passively move the patient's head into left side bending 4) passively move the patient's head into right side bending

3) passively move the patient's head into left side bending

August 10, 2020 An infant with developmental dysplasia of the hip is referred to physical therapy. Which of the following devices would the infant most likely use to reduce a dislocated hip? 1) adductor splint 2) abductor pillow 3) pavlik harness 4) scottish-rite orthotic

3) pavlik harness

March 11, 2020 A patient without a cardiac history describes the sudden onset of substernal pain that has radiated to the subclavicular area over the last five hours. The patient indicates that the knifelike pain increases with inspiration and movement and decreases when seated and leaning forward. The therapist identifies a pericardial friction rub through auscultation. The most likely diagnoses is: 1) myocardial infarction 2) angina 3) pericarditis 4) myocarditis

3) pericarditis

June 29, 2020 A physical therapist treats a patient with right homonymous hemianopsia. The patient is positioned in sitting in the middle of a small table. When reaching for objects placed at different locations around the table, what objects would the patient have the most difficulty retrieving? 1) place at the top center of the table 2) placed to the left side of the table 3) placed to the right side of the table 4) place at the bottom center of the table

3) placed to the right right of the table

Nov 1, 2019 A physical therapist treats a patient with right homonymous hemianopsia. The patient is positioned in sitting in the middle of a small table. When reaching for objects placed at different locations around the table, the patient would have the most difficulty retrieving objects: 1) placed at the top center of the table 2) placed to the left side of the table 3) placed to the right side of the table 4) placed at the bottom center of the table

3) placed to the right side of the table

August 31, 2020 A patient reports tp physical therapy one week status post transtibial amputation. Which sleeping position would be the most appropriate to prevent a knee and hip contracture? 1) supine with pillow placed under the knees 2) sidelying on the side of the residual limb with a pillow between the legs 3) prone lying without the use of any pillows 4) side-lying on the non-amputated side without a pillow between the legs

3) prone lying without the use of any pillows

February 24, 2021 A physical therapist examines a patient following a traumatic crush injury to the forearm. The patient displays a wrist drop deformity and is unable to strongly grasp objects with their hand. Which nerve was most likely affected? 1) median nerve 2) ulnar nerve 3) radial nerve 4) musculocutaneous nerve

3) radial nerve

May 31, 2020 An individual with an injury to which structure would have the most difficulty performing wrist extension? 1) median nerve 2) brachioradialis muscle 3) radial nerve 4) anconeus muscle

3) radial nerve

November 24, 2020 How does a pulmonary embolus affect the ventilation perfusion relationship in the alveolar capillaries? 1) ratio is normal; 0.8 2) ratio decreases to less than 0.8 3) ratio increases to more than 0.8 4) the ratio is 0.0

3) ratio increases to more than 0.8

Jan 15, 2020 A physical therapist completes lower extremity range of motion activities with a patient status post spinal cord injury.While performing PROM, the therapist notices the patient's urine is extremely dark and has a distinctive foul smelling odor. Which of the following is the most appropriate action for the therapist to take? 1) check for other signs of infection 2) report observation to physician 3) report observation to nurse 4) educate the patient on importance on proper hydration

3) report observation to nurse

Dec 7, 2019 A physical therapist employed in a rehabilitation hospital utilizes a variety of transfer techniques to move patients of various functional abilities. Which type of transfer would not be classified as dependent? 1) sliding transfer 2) hydraulic lift 3) sliding board transfer 4) two-person lift

3) sliding board transfer

January 20, 2021 A physical therapist observes the standing posture of a patient from a lateral view. If the patient has normal postural alignment, which of the following anatomical reference points would be most appropriate for where the plumb line would fall? 1) posterior to the lobe of the ear 2) slightly anterior to the center of the hip joint 3) slightly anterior to a midline of the knee 4) slightly posterior to the lateral malleolus

3) slightly anterior to a midline of the knee

November 3, 2020 A physical therapist discusses the importance of a proper diet with a patient diagnosed with CHF. Which of the following substances would most likely be restricted in the patient's diet? 1) HDL 2) LDL 3) sodium 4) triglycerides

3) sodium

August 11, 2020 A physical therapist records the end-feel associated with forearm supination as firm in the medical record. Which of the following is not consistent with an end-feel categorized as firm? 1) muscular stretch 2) capsular stretch 3) soft tissue approximation 4) ligamentous stretch

3) soft tissue approximation

MAy 29, 2020 A physical therapist observes a patient complete a standing squat. During the squat the therapist notes that the patient is unable to maintain their heels in contact with the ground. Based on this finding, what is most likely shortened? 1) gastroc 2) popliteus 3) soleus 4) tib anterior

3) soleus

Nov 9, 2019 A physical therapist observes a patient completing a standing squat shown in the image. During the squat the therapist notes that the patient is unable to maintain their heels in contact with the ground. This finding is most likely due to shortness of the: 1) gastrocnemius 2) popliteus 3) soleus 4) tibalis anterior

3) soleus

March 27, 2020 A physical therapist treats a patient with RA. Assuming the patient is in the active inflammatory phase, which intervention would be the most desirable? 1) Grade IV mobs 2) Active stretching 3) splinting for joint protection 4) aerobic endurance training

3) splinting for joint protection

February 9, 2021 A physical therapist assesses a pressure injury and identifies the presence of tunneling. With which pressure injury stage is this observation most likely to be indentified? 1) stage 1 and 3 2) stage 2 and 4 3) stage 3 and 4 4) stage 2 and unstageable

3) stage 3 and 4

Jan 6, 2020 A physician informs a patient that recent testing reveals that her breast cancer has spread to tissues adjacent to the primary tumor. The cancer cells have fully infiltrated the lymph nodes in this area, however, have not spread to other areas of the body. This description is most consistent with which stage of cancer? 1) stage I 2) stage II 3) stage III 4) stage IV

3) stage III

Nov 15, 2019 A physical therapist treats a patient who has been on bed rest for two weeks and is demonstrating signs of orthostatic hypotension. Which of the following interventions would the therapist be least likely to use? 1) elastic stockings 2) tilt table 3) standing frame 4) abdominal binder

3) standing frame

Dec 30, 2019 A physical therapist gathers a variety of equipment prior to administering a series of sensory tests. Which form of sensation would most likely be examined by utilizing a comb? 1) graphesthesia 2) vibration 3) stereognosis 4) barognosis

3) stereognosis

August 3, 2020 A 73 year old male patient receiving outpatient physical therapy begins to experience acute angina. The patient indicates he uses nitroglycerin to alleviate the angina. What is the most appropriate mode of administeration? 1) oral 2) buccal 3) sublingual 4) topical

3) sublingual

Feb 22, 2020 A physical therapist reads in a medical chart that a patient is taking a medication via enteral administration. Which of the following methods of medication administration would be considered a type of enteral administration? 1) intrathecal 2) subcutaneous 3) sublingual 4) transdermal

3) sublingual

April 2, 2021 A patient with osteogenesis imperfecta is examined by a PT due to reports of upper extremity pain. The therapist decides to performa series of special tests as part of the examination. Given the diagnosis, which of the following special tests would most likely be positive? 1) cervical foraminal compression test 2) adson maneuver 3) sulcus sign 4) lift off sign

3) sulcus sign

May 10, 2020 A patient with osteogenesis imperfecta is examined by a physical therapist due to reports of upper extremity pain. The therapist decides to perform a series of special tests as part of the examination. Given the diagnosis, which of the following special tests would most likely be positive? 1) cervical foraminal compression test 2) adson's 3) sulcus sign 4) lift off sign

3) sulcus sign

September 27, 2020 A patient 2 days s/p transfemoral amputation demonstrates decreased strength and generalized deconditioning. Which of the following positions should be utilized when instructing the patient to wrap their residual limb? 1) sidelying 2) standing 3) supine 4) prone

3) supine

November 17, 2020 A physical therapist notices that a patient s/p thoracic surgery has significantly altered his breathing pattern. The therapist hypothesizes that the alteration in breathing may be exacerbated due to pain from a sternontomy. What is the most appropriate intervention to treat and prevent atelectasis? 1) pursed-lip breathing 2) inspiratory muscle training 3) sustained maximal inspiration 4) postural drainage

3) sustained maximal inspiration

August 7, 2020 A physical therapist administers the Berg Balance Scale with a patient s/p posterior right CVA prior to d/c form the rehab facility. The patient scored a total of 39. What should the therapist conclude? 1) patient will require a low burden of care once d/c home 2) patient will have difficulty independently reaching for objects 3) the patient is at significant risk for multiple falls 4) patient will require assistance during ambulation

3) the patient is at significant risk for multiple falls

August 15, 2020 A physical therapist decides to work on balance with a patient in the modified plantigrade position. Which of the following is a benefit of using this position during treatment? 1) the patient must rely on only the LE for balance reactions 2) the position decreases extensor tone in the patient's wrists and elbows 3) the position uses a wider bass of support than does standing 4) the position increases flexion range of motion at the fingers

3) the position uses a wider bass of support than does standing

March 10, 2020 A patient recently diagnosed with end-stage renal disease begins hemodialysis treatments at an outpatient facility. Which frequency and duration of dialysis is the most typical? 1) one time a week for 5 hours 2) two times a week for 3 hours 3) three times a week for 4 hours 4) five times a week for 2 hours

3) three times a week for 4 hours

June 6, 2020 A physical therapist transfers a patient in a wheelchair down a curb with a forward approach. Which of the following actions would be the most appropriate? 1) have the patient lean forward 2) have the wheelchair brakes locked 3) tilt the wheelchair backwards 4) position yourself in front of the patient

3) tilt the wheelchair backwards

June 17, 2020 A physical therapist examines a patient with a past medical history that includes a transient ischemic attack. What variable most differentiates a TIA from a stroke? 1) presence of an aura 2) magnitude of the initial symptoms 3) time for resolution of ischemia 4) extent of speech and vision problems

3) time for resolution of ischemia

March 8, 2020 A physical therapist instructs a patient with a unilateral amputation to ascend and descend stairs. Which amputation level would you expect to have the most difficulty performing the described task? 1) transmetatarsal 2) transtibial 3) transfemoral 4) Syme's

3) transfemoral

June 14, 2020 A physical therapist examines a patient with a C6 spinal cord injury. Which muscle would not be innervated based on the patient's level of injury? 1) biceps 2) deltoid 3) triceps 4) diaphragm

3) triceps

March 3, 2020 A physical therapist palpates the bony structures of the wrist and hand. Which of following structures would not be identified in the distal row of carpals? 1) capitate 2) hamate 3) triquetrum 4) trapezoid

3) triquetrum

Jan 14, 2020 A patient three months post myocardial infarction has a follow-up electrocardiogram at his physician's office. The electrocardiogram shows three consecutive premature ventricular contractions with the absence of P waves. This description is most consistent with which of the following dysrhythmias? 1) atrial flutter 2) ventricular asystole 3) ventricular tachycardia 4) third-degree heart block

3) ventricular tachycardia

Feb 5, 2020 A physical therapist uses a subjective pain scale to assess pain intensity in patient with MS. The pain scale consists of a 10 cm line with each end anchored by one extreme of perceived pain intensity. The patient is asked to mark on the line the point that best describes their present pain level. This type of scale is best termed: 1) descriptor differential scale 2) verbal rating scale 3) visual analog scale 4) numerical rating scale

3) visual analog scale

May 19, 2020 A patient presents with deep partial-thickness burns to 65% of the total body surface area following a motor vehicle accident. Due to the extent of the burns pig skin is used as a temporary graft during the healing process. What type of graft has been used on this patient? 1) mesh 2) allograft 3) xenograft 4) sheet graft

3) xenograft

November 25, 2020 A physical therapist is working on balance activities and gait with a patient who has Parkinson's disease. The therapist is attempting to schedule home therapy visits based on the timing of the patient's levodopa medication. The therapist should coordinate home visits with which of the following dosing schedule to maximize the effects of treatment? 1) 30 minutes following lunch dose 2) 1 hour following the lunch dose 3) 30 minutes following the breakfast dose 4) 1 hour following the breakfast dose

4) 1 hour following the breakfast dose

July 19, 2020 A medical record indicates that a child's oral temperature was recorded as 100.6 degrees F. Which value would be most expected if the patient's rectal temperature was taken? 1) 98.6 2) 99.6 3) 100.6 4) 101.6

4) 101.6

March 10, 2021 A physical therapist administers the Modified Ashworth Scale to a patient with a suspected neurological injury. What is the most accurate description of the starting position when testing the elbow flexors? 1) full extension of the elbow 2) 45 degrees of elbow flexion 3) 90 degrees of elbow flexion 4) 145 degrees of elbow flexion

4) 145 degrees of elbow flexion

August 26, 2020 A patient using a transtibial prothesis demonstrates the ability to ambulate at both a fast cadence and a slow cadence while ambulating down a hallway. Based on this observation, what is the most appropriate K-level to assign to this patient? 1) 0 2) 1 3) 2 4) 3

4) 3

July 20, 2020 A physical therapist instructs a patient how to use a recumbent bicycle as part of a cardiovascular conditioning program. Which body mass index value would provide the most appropriate justification for using this type of equipment? 1) 15 2) 19 3) 24 4) 31

4) 31

December 5, 2020 A physical therapist conducts an inservice on Alzheimer's disease. Who is the relative risk of this disease greatest with? 1) 46 male 2) 42 female 3) 81 male 4) 83 female

4) 83 female

April 5, 2021 A PT intitiates an exercise program for a pt rehabing from cardiac surgery. During the treatment session, the therapist monitors the patient's O2 sat rate. Which of the following would be most representative of a normal oxygen saturation rate? 1) 82% 2) 87% 3) 92% 4) 97%

4) 97%

July 24, 2020 A physical therapist requests that a physician supervise a submaximal graded exercise test. Which scenario would provide the best support for the therapist's decision? 1) A 48-year-old who is asymptomatic with no known risk factors for coronary artery disease 2) A 36-year-old male who is asymptomatic with no known risk factors for coronary artery disease 3) A 42-year-old female who is asymptomatic with 1 known risk factors for coronary artery disease 4) A 28-year-old male who is symptomatic with no known risk factor for coronary artery disease

4) A 28-year-old male who is symptomatic with no known risk factor for coronary artery disease

September 26, 2020 A physical therapist treats a 14-year old child with a pulmonary condition. The therapist teaches the patient various postural drainage positions. The child most likely has what medical condition? 1) idiopathic pulmonary fibrosis 2) pulmonary edema 3) pneumonia 4) CF

4) CF

March 9, 2020 A physical therapist performs passive range of motion to a patient in the intensive care unit with severe congestive heart failure. The patient displays cycles of deep breathing following by shallow breathing with periods of apparent apnea. This breathing pattern is most characteristic of: 1) Kussmaul's 2) eupenea 3) othropnea 4) Cheyne-stokes

4) Cheyne-stokes

December 21, 2020 A physical therapist instructs a patient in an upper extremity proprioceptive neuromuscular facilitation pattern by telling the patient to begin by grasping an imaginary sword positioned on their left hip. What PNF pattern is this? 1) D1 ext 2) D1 flex 3) D2 ext 4) D2 flex

4) D2 flex

June 28, 2020 A physical therapist treats a patient with suspected bicipital tendon pathology. Which special test would be least likely to confirm the suspected diagnosis? 1) Ludington's test 2) Speed's test 3) Yergason's test 4) Drom arm test

4) Drom arm test

December 6, 2020 A physical therapist works with a patient on proprioceptive and balance activities due to a history of recent falls. The patient informs the therapist that they have recently been diagnosed with optic neuritis. Which of the following medical conditions should the therapist most suspect might be the cause of the patient's clinical presentation? 1) Parkinson's 2) Guillain-Barre 3) Myasthenia gravis 4) MS

4) MS

June 20, 2020 A physical therapist is treating a patient two weeks post RTC repair. Which of the following interventions is the most appropriate? 1) light progressive shoulder exercises with resistive tubing 2) grade III joint mobs 3) AROM up to 90 degrees of abduction 4) PROM up to 10 degrees of ER

4) PROM up to 10 degrees of ER

Dec 26, 2019 A physical therapist prescribes a home exercise program for a patient referred to physical therapy after being diagnosed with a disease process. The exercise program emphasizes large amplitude movements. This type of exercise program would be most beneficial for which of the following conditions? 1) Alzheimer's 2) Huntington's 3) ALS 4) Parkinson's

4) Parkinson's

March 22, 2021 A patient is referred to an outpatient ortho clinic with complaints of bilateral knee joint pain. During the exam, the PT notes a rash over the patient's cheeks and nose in the shape of a butterfly. What is most likely the diagnosis? 1) fibromyalgia 2) sarcoidosis 3) systemic sclerosis 4) SLE

4) SLE

Nov 3, 2019 A physical therapist treats a patient with a medical history that includes Klinefelter syndrome. What is the exact etiology of this genetic condition? 1) Autosomal dominant disorder 2) Autosomal trisomy disorder 3) Partial deletion disorder 4) Sex chromosome aneuploidy disorder

4) Sex chromosome aneuploidy disorder

March 21, 2020 A patient recieves surgical dischare instructions that include the use of an incentive spirometer and lower extremity compression stockings. Which of the following surgical procedures would be the most likely to necessitate the use of this medical equipment? 1) achilles tendon repair 2) lateral ankle ligament reconstruction 3) knee meniscectomy 4) TKA

4) TKA

April 1, 2021 During palpation, a PT determines that the spine of a patient's scapula is level with the spinous process of T2. Based on this finding, which exercise is a therapist most likely to prescribe? 1) shoulder shrugs with dumbbells 2) horz add with resistance band 3) serratus punches with DB 4) UT stretch

4) UT stretch

March 14, 2021 A physical therapist reviews a patient's medication list as part of an examination. The patient is ten days post left THA. Which medication would most likely prohibit the patient from driving? 1) Celebrex (Celecoxib) 2) Mobic (Meloxicam) 3) Medrol (methylprednisolone) 4) Ultram (Tramadol)

4) Ultram (Tramadol)

December 27, 2020 A physical therapist assesses the saccadic movements of a patient with a mild TBI and notes that the patient has difficulty shifting their gaze laterally, though the other movements occur normally. Which cranial nerve is most likely impaired? 1) II 2) III 3) IV 4) VI

4) VI

July 23, 2020 A physical therapist assesses the saccadic movements of a patient with a mild traumatic brain injury and notes that the patient has difficulty shifting their gaze laterally, though the other movements occur normally. This impairment most likely denotes damage to what cranial nerve? 1) II 2) III 3) IV 4) VI

4) VI

June 8, 2020 A physical therapist performs an upper limb neural tension test to bias the median nerve. Upon completing the sequence of movements for the upper extremity, the therapist utilizes lateral flexion of the neck to the contralateral side to sensitize the maneuver. Which of the following questions provides the best method to determine the patient response to this manuever? 1) Do your symptoms decrease with this movement 2) Do your symptoms increase with this movement 3) Do your symptoms stay the same with this movement 4) What happens to your symptoms during this movement

4) What happens to your symptoms during this movement

Dec 15, 2019 A physical therapist reads in the medical record that a patient recently experienced a first-degree atrioventricular heart block. Which of the following findings would the therapist most likely expect to observe when analyzing the patient's electrocardiogram strip? 1) an absent QRS complex 2) an elevated ST segment 3) a wide QRS complex 4) a prolonged PR interval

4) a prolonged PR interval

April 23, 2020 A patient with osteoporosis is referred to physical therapy following the diagnosis of a healing T10 vertebral compression fracture. Which of the following exercises should be avoided as part of the patient's plan of care? 1) seated shoulder horz abd with a resistance band 2) push-ups from a counter top 3) planks from the knees and forearms 4) abdominal crunches on the floor

4) abdominal crunches on the floor

Feb 1, 2020 A patient with acute back pain is given a transcutaneous electrical nerve stimulation unit to use at home. The physical therapist provides detailed instructions on the care and use of the unit. Which of the following activities is not the responsibility of the patient? 1) modulate the current intensity 2) application of new electrodes 3) change the battery 4) alter the pulse rate and width

4) alter the pulse rate and width

July 25, 2020 A physical therapist works with a patient six weeks following THA (posterolateral approach). The patient's medical history includes Grave's disease. Which intervention should the therapist avoid when treating the patient? 1) supine resisted hip ER 2) standing hip abd with an ankle weight 3) stair training 4) ambulation in a warm therapy pool

4) ambulation in a warm therapy pool

September 21, 2020 A patient ambulating in the physical therapy gym suddenly grabs the physical therapist's arm and indicates he feels faint. What is the most immediate action to take? 1) assess pulse 2) ask the patient if they have ever fainted 3) loosen tight clothes 4) assist patient to sitting position

4) assist patient to sitting position

November 23, 2020 A patient post cardiac surgery is monitored using an arterial line. The primary purpose of an arterial line is to monitor which of the following measures? 1) right arterial pressure 2) heart rate and oxygen saturation 3) pulmonary artery pressure 4) blood pressure

4) blood pressure

Feb 26, 2020 A patient with a stage III pressure injury over the ischial tuberosity presents with copious amounts of seropruriulent drainage from the wound bed. The physical therapist is concerned about maceration of the periwound tissues. Based on this information, which of the following wound dressings would be the most appropriate choice for this patient? 1) transparent films 2) impregnated gauze 3) hydrocolloids 4) calcium alginates

4) calcium alginates

Jan 23, 2020 A physical therapist examines a patient recently diagnosed with a spinal cord syndrome secondary to a tumor. The patient has slowly developed difficulty with bowel and bladder function, has weakness in the bilateral lower extremities, and presents with a "saddle area" sensory impairment, The most likely diagnosis is: 1) central cord syndrome 2) conus medullaris syndrome 3) anterior spinal artery syndrome 4) cauda equina syndrome

4) cauda equina syndrome

May 24, 2020 A physical therapist examines a patient who has an entrance wound on the top of their head and an exit wound on the right groin secondary to an electrical burn obtained at work. The therapist observes the patient exhibiting gaze-evoked nystagmus and intention tremors. Which area of the brain was most likely damaged by this burn. 1) temporal lobe 2) occipital lobe 3) pons 4) cerebellum

4) cerebellum

Dec 1, 2019 A patient has an echocardiogram following an MI. The echocardiogram shows damage to the posterolateral wall of the left ventricle. Which coronary artery was most likely occluded based on the involved area of the heart? 1) right coronary artery 2) posterior descending artery 3) left anterior descending artery 4) circumflex artery

4) circumflex artery

Jan 24, 2020 A physical therapist reads in a patient's medical record that the ankle-brachial index was measured as 0.36. Which scenario is most indicative of the obtained ABI value? 1) paresthesias in the feet and toes 2) weakened calf muscles 3) intermittent claudication during exercise 4) claudication pain at rest

4) claudication pain at rest

Feb 7, 2020 A physical therapist conducts an examination on a patient diagnosed with Parkinson's disease. Which of the following clinical findings would the therapist expect to identify? 1) aphasia 2) ballistic movements 3) severe muscle atrophy 4) cogwheel rigidity

4) cogwheel rigidity

March 3, 2020 A physical therapist works with a patient two days post TKA. The patient is currently taking oxycodone for acute pain control following surgery. Based on the prescribed medication, which of the following supplements would most likely also be given to this patient? 1) niacin 2) glucosamine 3) folic acid 4) colace

4) colace

September 20, 2020 A patient presents with signs and symptoms that are consistent with stage II lymphedema of the right lower extremity. Assuming the patient has progressed to the maintenance phase of treatment, which of the following compression therapy interventions would be most appropriate? 1) short-stretch bandaging during the day 2) long-stretch bandaging during the night 3) compression garment during the night 4) compression garment during the day

4) compression garment during the day

October 28, 2020 A physical therapist places a patient in supine with their legs elevated after the patient experienced the signs and symptoms of orthostatic hypotension during a treatment session. A patient with which of the following conditions would most likely experience difficulties in this recovery period? 1) lumbar spinal stenosis 2) chronic venous insufficiency 3) Grave's disease 4) congestive heart failure

4) congestive heart failure

December 13, 2020 A patient status post Achilles tendon repair is examined in physical therapy. The physician referral includes a very specific post-operative protocol. Which action would be most appropriate if the therapist plans on deviating from the established protocol? 1) secure the patient's surgical report 2) complete a thorough exam 3) carefully document any modification 4) contact the referring MD

4) contact the referring MD

January 5, 2021 A patient in the physical therapy gym suddenly grasps their throat and begins to cough. The physical therapist, recognizing the potential for an airway obstruction, should take which of the following actions? 1) attempt to ventilate 2) administer abdominal thrusts 3) perform a quick finger sweep of the mouth 4) continue to observe, but do not interfere

4) continue to observe, but do not interfere

January 31, 2021 A patient with atrial flutter suddenly becomes extremely lightheaded while exercising and appears to lose consciousness. Which of the following physiologic responses best explains the change in the patient's status? 1) increased filling of the ventricles 2) increased system congestion 3) decreased atrial depolarization 4) decreased blood traveling to the brain

4) decreased blood traveling to the brain

October 21, 2020 A patient status post right transfemoral amputation is prescribed a standard wheelchair with recommendation for posterior rear wheel axle placement. What does the positioning result in? 1) decreased rear stability 2) decreased rolling resistance 3) decreased turning radius 4) decreased ease of performing wheelies

4) decreased ease of performing wheelies

March 25, 2021 A physical therapist employed in an acute care hospital treats a patient that sustained a comminuted fracture of the humeral head in a MVA. Prior to initiating the daily treatment session, the therapist reads in the medical record that the patients was diagnosed with septic arthritis. What is the most appropriate therapist action given this additional information? 1) continue with plan of care 2) incorporate superficial heat for pain 3) utilize mobilization techniques to maintain range of motion 4) discontinue PT until patient is medically cleared

4) discontinue PT until patient is medically cleared

February 23, 2021 A physical therapist examines a patient diagnosed with spondylolisthesis. The patient is a 12-year-old competitive gymnast who reports occasional low back pain that has not significantly impacted her training schedule. Diagnostic imaging reveals a 50% anterior slippage of the L5 vertebrae. What is the most appropriate activity modification recommendation for this patient? 1) continue with gymnastic training and competition as symptoms allow 2) continue a limited training routine until physical therapy is completed 3) defer gymnastic training and competition until physical therapy is completed 4) discontinue gymnastic training and competition

4) discontinue gymnastic training and competition

January 15, 2021 A patient sustains a chemical burn on the cubital area of the elbow. What position would be the most appropriate for splinting of the involved upper extremity? 1) elbow flexion and forearm pronation 2) elbow flexion and forearm supination 3) elbow extension and forearm pronation 4) elbow extension and forearm supination

4) elbow extension and forearm supination

September 29, 2020 A physical therapist concludes that it is necessary to elongate the long head of the triceps brachii as part of a passive range of motion assessment of the elbow. Which positioning of the arm would assist the therapist to meet the stated objective? 1) elbow extension and shoulder extension 2) elbow extension and shoulder flexion 3) elbow flexion and shoulder extension 4) elbow flexion and shoulder flexion

4) elbow flexion and shoulder flexion

January 1, 2020 A physical therapist treats a child with an obstetric brachial plexus injury. Which of the following interventions would be least useful to improve the child's functional status? 1) active movement 2) range of motion 3) splinting 4) electrical stimualtion

4) electrical stimualtion

May 11, 2020 A physician examines a child with suspected osteogenesis imperfecta due to a history of multiple fractures and bowing of the legs. Which of the following diagnostic tools would be most useful to help confirm the suspected diagnosis? 1) positron emission tomography 2) fluoroscopy 3) lumbar puncture 4) genetic testing

4) genetic testing

July 13, 2020 A physical therapist employed in an acute care hospital works with a patient with a known latex allergy. Which of the following items would be least likely to contain latex? 1) gloves 2) surgical masks 3) intravenous tubing 4) gowns

4) gowns

February 14, 2021 A 45-year-old female with RA has complaints of global neck and shoulder pain. She has been on prednisone since she was diagnosed at the age of 30 to help slow the progression of the disease. Which of the following interventions would be unlikely to be included as a component of the patient's care plan? 1) open chain strengthening with cuff weights 2) push-ups from a counter top 3) aquatic therapy 4) grade 3 spinal mobilizations

4) grade 3 spinal mobilizations

July 18, 2020 A 45-year-old female with rheumatoid arthritis has complaints of global neck and shoulder pain. She has been on prednisone since she was diagnosed at the age of 30 to help slow the progression of the disease. Which of the following interventions would be unlikely to be included as a component of the patient's care plan? 1) open chain strengthening with cuff weights 2) push ups from counter top 3) aquatic therapy 4) grade 3 thoracic spinal mobilizations

4) grade 3 thoracic spinal mobilizations

May 13, 2020 A 25-year-old patient with osteogenisis imperfecta is treated in physical therapy for a general conditioning program. Which of the following comorbidities would most likely be expected based on the diagnosis? 1) artherosclerosis 2) cirrhosis 3) cortical blindness 4) hearing loss

4) hearing loss

Jan 16, 2020 A patient with a C4 spinal cord injury in an acute care hospital prepares to transfer from the bed to their wheelchair. The patient's BMI is 38.4. Which of the following transfer techniques would be the most appropriate for this patient? 1) dependent squat pivot transfer 2) sliding board transfer 3) two-person lift 4) hydraulic lift

4) hydraulic lift

September 22, 2020 A physical therapist treats a patient following knee arthroscopy after observing a limitation in knee flexion range of motion when the patient was positioned in hook-lying. The therapist uses a contract-relax stretching technique to improve the patient's range of motion. Which clinical scenario would most benefit from this intervention? 1) stiffness of the joint capsule 2) decreased flexibility of the rectus femoris 3) bony obstruction due to arthritic joint surfaces 4) hypertonicity of the quads

4) hypertonicity of the quads

Nov 26th, 2019 A physical therapist reviews a chart prior to treating a patient in the hospital coronary care unit. A recent entry indicates that the patient is taking an angiotension-converting enzyme (ACE) inhibitor. Which of the following side effects is common the ACE inhibitors and would most impact the therapist performing transfer training with this patient? 1) bradycardia 2) dehydration 3) hypokalemia 4) hypotension

4) hypotension

April 8, 2020 A physical therapist treats a patient with osteoporosis for a generalized strengthening program. Based on this diagnosis, which of the following medications would most likely be included in the patient's medication list? 1) esomeprazole (nexium) 2) methotrexate (rheumatrex) 3) metformin (riomet) 4) ibandronate (boniva)

4) ibandronate (boniva)

January 28, 2021 A physical therapist completes a two-point discrimination sensory assessment on the right hand of a patient who has a peripheral nerve injury. Which of the following instructions is the most appropriate for the therapist to ask the patient to indicate during the test? 1) the specific location where the stimulus is felt 2) when two points are felt 3) when the stimulus is first felt 4) if one or two points are felt

4) if one or two points are felt

April 19, 2020 A patient diagnosed with postmenopausal osteoporosis is referred to physical therapy to work on general strengthening program. Which pathophysiology is responsible for the development of this condition? 1) increased osteoblast and osteoclast activity 2) decreased osteoblast and osteoclast activity 3) increase in osteoblast, decrease in osteoclast 4) increase in osteoclast, and decrease in osteoblast

4) increase in osteoclast, and decrease in osteoblast

January 10, 2021 A physical therapist decides to discontinue a patient treatment session due to a suspected pulmonary embolism. Which of the following clinical findings best supports the therapist hypothesis? 1) partial pressure of oxygen of 85 2) RR of 14 3) RHR 100 4) increased cough with presence of hemoptysis

4) increased cough with presence of hemoptysis

February 5, 2021 A physical therapist treats a patient following knee arthroscopy after observing a limitation in knee flexion range of motion when the patient was positioned in hooklying. The therapist uses a contract-relax stretching technique to improve the patient's ROM. Which clinical scenario would most benefit from this intervention? 1) stiffness in the joint capsule 2) decreased flexibility of the rectus femoris 3) bony obstruction due to arthritic joint surfaces 4) increased quad muscle tone

4) increased quad muscle tone

January 22, 2021 During a treatment session, a patient makes a culturally insensitive remark that the PT feels is offensive. Which of the following actions is the most appropriate for the therapist to take? 1) document incident in medical record 2) transfer patient to another PT's schedule 3) recommend d/c 4) inform patient the remark was offensive and continue treatment

4) inform patient the remark was offensive and continue treatment

January 17, 2021 A physical therapist initiates an exercise program for a patient who has a lower extremity injury. Which of the following parameters is the single most important factor in an exercise program designed to increase muscular strength? 1) recovery time between exercise sets 2) number of repetitions per set 3) duration of the exercise session 4) intensity of the exercise

4) intensity of the exercise

April 24, 2020 A physical therapist examines a patient who is one week post calcaneofibular ligament sprain. The patient reports experiencing a similar injury multiple times over the past five years. When performing the examination, which motion of the ankle and foot complex would likely be the most provocative for this patient? 1) PF 2) DF 3) eversion 4) inversion

4) inversion

November 13, 2020 A physical therapist works with a patient diagnosed with ACL insufficiency. The physician referral specifies closed kinematic chain rehabilitation. Which exercise would not be appropriate based on the order? 1) exercise on a stair machine 2) limited squats to 45 degrees 3) walking backwards on a treadmill 4) isokinetic knee ext and flex

4) isokinetic knee ext and flex

Nov 21, 2019 If continuation of standing and walking is indicated for an adolescent with Duchenne muscular dystrophy, the physical therapist should consider which of the following equipment options? 1) ankle-foot orthosis and bilateral canes 2) ankle-foot orthoses and bilateral Lofstrand crutches 3) knee-ankle-foot orthoses and bilateral canes 4) knee-ankle-foot orthoses and walker

4) knee-ankle-foot orthoses and walker

November 19, 2020 A K2 level ambulator with a unilateral transfemoral prothesis is learning to ambulate up a steep incline (20 degrees). What technique is the most appropriate to navigate steep inclinations? 1) lead with prosthetic limb and ambulate with a reciprocal gait pattern 2) lead with the contralateral limb and ambulate with a reciprocal gait pattern 3) lead with prosthetic limb and ambulate with a step-to gait pattern 4) lead with the contralateral limb and ambulate with a step-to gait pattern

4) lead with the contralateral limb and ambulate with a step-to gait pattern

Dec 17, 2019 A physical therapist works with a patient that required a mechanical ventilator during an acute care hospitalization. If the patient continues to experience dyspnea due to a weakened immune diaphragm post mechanical ventilation, which of the following patient positions is the most desirable to provide relief from the dyspnea? 1) lying supine with the lower extremities elevated 2) trendelenburg positioning in sidelying 3) semi-fowler positioning 4) leaning forward in sitting with upper extremity support

4) leaning forward in sitting with upper extremity support

March 13, 2021 A physical therapist elects to administer a submaximal graded exercise test to a patient prior to prescribing an aerobic training program. Which statement is a valid assumption of a submaximal exercise test? 1) steady HR is obtained at progressively decreasing work rates 2) inverse relationship between HR and work rate 3) maximal HR for a given chronological age is variable 4) max oxygen consumption at a given work rate is the same for everyone

4) max oxygen consumption at a given work rate is the same for everyone

Dec 27, 2019 A note in a patient's medical record indicates that they are taking oral potassium chloride for an acid-base disorder. Which acid-base disorder is most likely based on the prescribed medication? 1) respiratory acidosis 2) respiratory alkalosis 3) metabolic acidosis 4) metabolic alkalosis

4) metabolic alkalosis

April 9, 2021 A PT performs girth measurements on a pt rehabing from knee surgery. The therapist takes measurements 5 cm and 10 cm above the superior pole of the patella with the patient in supine. The girth measurements are recorded as 32 cm and 37 cm on the right and 34 cm and 40 cm on the left. Which of the following conclusions can be made regarding the strength of the patient's quads? 1) right quad can produce greater force that the left 2) left quad can produce greater force that the right 3) equal force from both quads 4) not enough info

4) not enough info

February 1, 2021 A physician orders a nasogastric tube for a patient admitted to the hospital. Which of the following indications does not accurately describe a potential use of the nasogastric tube? 1) administer medications directly into the gastrointestinal tract 2) obtain gastric specimens 3) remove fluid or gas from the stomach 4) obtain venous blood samples from the stomach

4) obtain venous blood samples from the stomach

Feb 18, 2020 A physical therapist transports a patient with multiple sclerosis to the gym for their treatment session. The patient is wheelchair dependent and uses a urinary catheter. When transporting the patient, where would be the most appropriate location to secure the collection bag? 1) in the patient's lap 2) attached to the IV pole 3) on the wheelchair 4) on the wheelchair cross brace beneath the seat

4) on the wheelchair cross brace beneath the seat

December 30, 2020 A physical therapist monitors a patient with a C6 spinal cord injury positioned on a tilt table. After elevating the tilt table to 30 degrees, the patient begins to complain of nausea and dizziness. The patient's blood pressure is measured as 70/35 mmHG. Which medical condition is most consistent with the patient's signs and symptoms? 1) spinal shock 2) postural hypotension 3) autonomic dysreflexia 4) orthostatic hypotension

4) orthostatic hypotension

January 4, 2021 A physical therapist monitors a patient with a C6 spinal cord injury positioned on a tilt table. After elevating the tilt table 3o degrees, the patient reports nausea and dizziness. The therapist measures the patient's blood pressure as 70/35 mmHG. The patient's signs and symptoms are most indicative of what condition? 1) spinal shock 2) postural hypertension 3) autonomic dysreflexia 4) orthostatic hypotension

4) orthostatic hypotension

June 27, 2020 An older adult describes posterior thoracic pain worsened with spinal flexion. They report no cough, no trouble breathing, and are a non-smoker. The patient has a past medical history of DM, hyperparathyroidism, and acid reflux, for which they take antacids as needed. What condition is this description most consistent with? 1) Paget's disease 2) lung neoplasm 3) costochondritis 4) osteoporotic fracture

4) osteoporotic fracture

October 2, 2020 A physical therapist discusses common cognitive and behavioral changes associated with stroke with family members of a patient who has right hemisphere damage. Which of the following phrases does not accurately describe the most typical patient presentation? 1) poor judgement 2) impulsive 3) quick 4) overly cautious

4) overly cautious

February 22, 2021 A patient is referred to physical therapy with a diagnosis of left shoulder impingement. During the exam, the therapist begins to suspect a systemic cause for the patient's pain. Which of the following symptoms would best support this hypothesis? 1) pain has been present for years 2) pain is alleviated with the use of cryotherapy 3) pain is aggravated by cervical rotation 4) pain remains unrelieved with rest

4) pain remains unrelieved with rest

April 5, 2020 A patient with recurrent lateral ankle instability undergoes an ankle reconstruction procedure. As part of the procedure, the surgeon feels that the ankle needs to be reinforced with a tendon autograft. Which of the following muscles would most likely serve as the source of the autograft tissue? 1) semitendinosus 2) gracilis 3) flexor hallucis longus 4) peroneus brevis

4) peroneus brevis

January 8, 2021 A patient returns from a physician visit and informs a physical therapist that they have decreased their systolic blood pressure by approximately 20 mmHg over the last two months. Which intervention would most likely be responsible for the decrease in systolic blood pressure? 1) dietary changes 2) limiting alcohol 3) activity level changes 4) pharmacological management

4) pharmacological management

June 11, 2020 A patient with a breathing dysfunction is diagnosed with a restrictive lung disease by their physician. Which of the following diagnosis would most likely be considered a restrictive lung disease? 1) asthma 2) bronchiectasis 3) emphysema 4) pleural effusion

4) pleural effusion

September 12, 2020 A physical therapist suspects that a patient being treated for shin splints may actually have chronic exertional compartment syndrome. Where should the therapist palpate to assess if perfusion to the foot has been affected? 1) anterior to the lateral malleolus 2) posterior to the lateral malleolus 3) anterior to the medial malleolus 4) posterior to the medial malleolus

4) posterior to the medial malleolus

January 26, 2021 A patient who has greater trochanteric bursitis completes an AROM exercise for hip flexion. Which direction does the femoral head slide on the acetabulum during hip flexion? 1) superiorly 2) inferiorly 3) anteriorly and superiorly 4) posteriorly and inferiorly

4) posteriorly and inferiorly

August 28, 2020 A physical therapist treats a patient in the ICU with significant fluid loss. What objective finding is most likely based on the patient's current medical status? 1) dependent edema 2) distending neck veins 3) orthopnea 4) postural hypotension

4) postural hypotension

April 8, 2021 A physical therapist working in a rehab hospital completes the Braden scale as a component of the patient examination. What is is most likely used for? 1) neuropathic ulcers 2) arterial insufficiency ulcers 3) venous insufficiency ulcers 4) pressure injuries

4) pressure injuries

Feb 13, 2020 A physical therapist working in a rehabilitation hospital completes the Braden scale as a component of patient examination. The Braden scale is most often used as a risk assessment tool for which of the following integumentary conditions? 1) neuropathic ulcers 2) arterial insufficiency ulcers 3) venous insufficiency ulcers 4) pressure injuries

4) pressure injuries

August 8, 2020 A physical therapist implements an exercise program that includes a 5 minute cool down period. What does the inclusion of a cool down period assist with? 1) increasing mitochondrial number and size 2) dilation of constricted capillaries 3) increasing maximum oxygen consumption 4) preventing pooling of blood in the extremities

4) preventing pooling of blood in the extremities

August 22, 2020 A physical therapist reviews the medical record of a patient diagnosed with lower motor neuron paralysis. What would the likely presentation include? 1) hypertonia or spasticity 2) normal nerve conduction study 3) fasciculations are absent 4) pronounced muscle atrophy

4) pronounced muscle atrophy

Jan 13, 2020 A physical therapist determines that a patient with a history of recurrent low back pain would benefit from frontal plane stabilization exercises that focus on activation of the quadratus lumborum. Which of the following exercises would best accomplish this objective? 1) balancing a rod on the back while performing an exercise in quadruped 2) single leg slides in supine position while maintaining a neutral spine 3) alternating leg lifts in a prone position while maintaining a neutral spine 4) propping up on the elbow in sidelying while lifting the pelvis off the matt (side plank)

4) propping up on the elbow in sidelying while lifting the pelvis off the matt (side plank)

A 52-year-old direct access patient is examined in physical therapy. The patient states that over the last three months he has experienced increasing neck stiffness and pain at night. He also communicates that he recently has several episodes of dizziness. The patient has a family history of cancer and has smoked two packs of cigarettes a day for over twenty years. The date of his last medical examination was ten years ago. The physical therapist's most appropriate action is to: 1) treat the patient conservatively and document any changes in the patient's status 2) inform the patient that he is not a candidate for physical therapy 3) refer the patient to an oncologist 4) refer the patient to his PCP

4) refer the patient to his PCP

Feb 3, 2020 A physical therapist observes a patient completing a low-level exercise test on a treadmill. Which of the following measurement methods would provide the physical therapist with an objective measurement of endurance? 1) facial color 2) facial expression 3) rating on perceived exertion scale 4) respiration rate

4) respiration rate

March 22, 2020 A physical therapist obsrves a patient completing a low-level exercise test on a treadmill. Which of the following measurement methods would provide the physical therapist with an objective measurement of endurance? 1) facial color 2) facial expression 3) RPE 4) respiration rate

4) respiration rate

February 3, 2021 A patient three weeks status post transtibial amputation exhibits intact skin integrity of the residual limb with normally appearing sutures. What is the most appropriate covering to apply to the residual limb to minimize the incidence of a knee contracture? 1) residual limb shrinker 2) prosthetic sock 3) elastic wrap bandages 4) rigid dressing

4) rigid dressing

June 18, 2020 A physical therapist performs an ULTT to bias the median nerve. Which of the following movements should the therapist perform first when performing this test? 1) abduction of the shoulder 2) lateral flexion of the neck 3) lateral rotation of the shoulder 4) shoulder girdle depression

4) shoulder girdle depression

December 23, 2020 A physical therapist observes that a patient pos traumatic brain injury exhibits toe drag during the swing phase of gait. Which of the following impairments should the therapist least expect to contribute to this gait deviation? 1) flaccidity of DF 2) inadequate DF ROM 3) weakness of DF 4) spasticity of DF

4) spasticity of DF

March 2, 2021 An older adult has difficulty communicating in physical therapy due to presbycusis. What intervention is the most appropriate to improve communication with the patient? 1) speak with a louder voice 2) speak in a higher tone 3) speak with a lower voice 4) speak in a lower tone

4) speak in a lower tone

November 20, 2020 A patient rehabbing from surgery discusses returning to their job as a foreman in a manufacturing facility. The job requires occasional lifting of objects up to 25 pounds and frequent trips up and downstairs, however, the patient's primary responsibility is supervising other employees. Which type of surgery would likely allow for the most rapid return to work? 1) ACL reconstruction 2) medium RTC repair 3) spinal fusion 4) subacromial decompression

4) subacromial decompression

January 7, 2021 A physical therapist notices significant atrophy of the infraspinatus muscle while treating a patient with shoulder pathology. This finding is most consistent with an injury to which of the following nerves 1) axillary 2) long thoracic 3) spinal acessory 4) suprascapular

4) suprascapular

Dec 18, 2019 A physical therapist assesses a patient with a diabetic foot ulcer. Using the Wagner Ulcer Grade Classification System, the therapist grades the ulcer as a score of 4. What would be the most appropriate intervention for this type of wound? 1) pulsed levage 2) total-contact casting 3) oral antibiotics 4) surgical amputation

4) surgical amputation

June 16, 2020 A physical therapist is asked to assist in administering a graded exercise test to a patient referred to a phase II cardiac rehabilitation program. Which of the following objective findings would most warrant the termination of the test? 1) diastolic BP rising to 100 mmHG 2) systolic BP rising to 200 mmHG 3) multifocal premature ventricular contractions 4) sustained ventricular tachycardia

4) sustained ventricular tachycardia

January 13, 2021 A physical therapist asks a patient to raise one hand to indicate that they felt a Semmes-Weinstein monofilament contacting their skin. Which situation would best support this type of testing procedure? 1) the patient does not speak english 2) the patient has ideomotor apraxia 3) the patient has diabetes mellitus 4) the patient has Broca's aphasia

4) the patient has Broca's aphasia

October 13, 2020 A physical therapist instructs a patient in a cardiac rehabilitation program to use a rate of perceived exertion scale to quantify the intensity of exercise. Which of the following provides the least support for the patient using rate of perceived exertion instead of heart rate? 1) patient's HR response to exercise is altered due to cardiac transplant 2) patient initiates an exercise-based program without a preliminary exercise test 3) patient has sensory deficits in the wrist and hand 4) the patient is taking a anticoagulant medication

4) the patient is taking a anticoagulant medication

February 12, 2021 A patient with a transtibial ampuation displays early knee flexion from midstance through preswing. What is a possible cause for this deviation? 1) the foot is set in neutral 2) the docket is set posterior in relation to the foot 3) the prothesis is too short 4) the socket is aligned in excessive flexion

4) the socket is aligned in excessive flexion

September 16, 2020 A physical therapist attempts to perform a series of special tests on a patient rehabilitating from a fall sustained while hiking. The therapist observes that the patient seems slightly unstable and experiences periods of significant dizziness when walking. Which of special test would likely be problematic for the patient to tolerate? 1) bounce home test 2) quadrant scouring test 3) talar tilt test 4) thessaly test

4) thessaly test

May 2, 2020 A patient with recurrent lateral ankle instability undergoes an ankle reconstruction procedure. The surgeon decides that it will be necessary to reinforce the ankle with a tendon graft. Which of the following factors would most likely have let the surgeon to that decision? 1) the patient's ankle joint has osteochondral defects 2) the patient has limited inversion ROM 3) the patient is unable to perform a SLS 4) this surgery is a revision of a previous reconstruction procedure

4) this surgery is a revision of a previous reconstruction procedure

April 6, 2020 A patient presents to physical therapy for treatment of a non-operative osteoporotic fracture. Which of the following structures would have been least likely affected? 1) distal radius 2) vertebral body 3) proximal femur 4) tibial plateau

4) tibial plateau

March 15, 2021 A patient presents to physical therapy for treatment of a non-operative osteoporotic fracture. Which of the following structures would have been least likely affected? 1) distal radius 2) vertebral body 3) proximal femur 4) tibial plateau

4) tibial plateau

Nov 28, 2019 A physical therapist positions a patient in supine and places the head and neck in maximal flexion, and then laterally rotates the head to the right side. While stabilizing the head in this position, the therapist then depresses the right shoulder girdle. Which right-sided muscle would be fully assessed for adequate length using this testing procedure? 1) SCM 2) levator scapulae 3) anterior scalene 4) upper trap

4) upper trap

October 5, 2020 A physical therapist progresses a patient involved in a phase II cardiac rehabilitation program through an established exercise protocol. The patient weighs 154 pounds and has progressed without difficulty through the program. The protocol indicates the patient should be performing activities requiring 3-4 METs. Which of the following activities is most appropriate to achieve this goal? 1) level walking at 1 mph 2) jogging at 5 mph 3) cycling at 10 mph 4) walking on a treadmill at 3 mph

4) walking on a treadmill at 3 mph

April 17, 2020 A physical therapist concludes that a patients exhibits dysmetria after performing a number of specific tests. Which test would be least helpful to confirm the therapist's conclusion? 1) drawing a circle or figure 8 2) pointing and past pointing 3) placing the feet on a marker while walking 4) walking with alterations in speed

4) walking with alterations in speed

April 26, 2020 A physical therapist listens to the lung sounds of a 56-year-old male with chronic bronchitis. The patient was admitted to the hospital two days ago after complaining of SOB and difficulty breathing. While performing auscultation, the therapist identifies distinct lung sounds with a high constant pitch during exhalation. What is this type of sound most consistent with? 1) crackles 2) rales 3) rhonchi 4) wheezes

4) wheezes

Oct 28, 2019 A physical therapist listens to the lung sounds of a 56-year-old male with chronic bronchitis. The patient was admitted to the hospital two days ago after complaining of shortness of breath and difficulty breathing. While performing auscultation, the therapist identifies distinct lung sounds with a high constant pitch during exhalation. This type of sound is most consistent with: 1) crackles 2) rales 3) rhonchi 4) wheezes

4) wheezes

May 21, 2020 A physical therapist reviews the medical record of a patient diagnosed with myasthenia gravis. A recent physician entry indicates that the patient is currently taking immunosuppressive medication. Which laboratory test should be the most frequently monitored based on the patients medication? 1) hematocrit 2) hemoglobin 3) platelet count 4) white blood cell count

4) white blood cell count

May 8, 2020 A patient recently admitted to the hospital with an acute illness is referred to physical therapy. During a scheduled treatment session, the patient asks what effect anemia will have on his ability to complete a formal exercise program. What is the most appropriate therapist response? 1) you may feel as though your muscles are cramping 2) you may experience frequent nausea 3) your aerobic capacity may be reduced 4) you may have a tendency to become fatigued

4) you may have a tendency to become fatigued

Oct 24, 2019 A physical therapist instructs a patient to change his wound dressing daily between weekly visits to the wound center. At the next visit, the patient reports that he instead changed the dressing only when it was soaked through in an effort to conserve supplies. As a result, the surrounding skin would most likely be described as: 1) Infected 2) Gangrenous 3) Macerated 4) Indurated

Macerated


Kaugnay na mga set ng pag-aaral

Chapter 5: The Expenditure Cycle

View Set

Psychosocial Mental Health Review Ch. 15-- Leik Content

View Set

Info and Network Security Chapter 14

View Set

Micro Chapter 9 - Quiz and Homework Questions

View Set

POLS 2306 - TX Gov - CH 12 Criminal Justice Policy in Texas

View Set

MCAT - CARS (Critical Analysis & Reasoning Skills)

View Set

A Tale of Two Cities Book the Third Quotes

View Set